You are on page 1of 147

Question ID 1594

Title Approach to misconduct of a doctor in hospital

Notes
Q Flag as important

Font Size: A A A

You, as medical doctors in a hospital, are assigned to review the files of doctors who have applied
for a certain position in that hospital and report the eligible doctors to the hospital manager for
further assessment. You realize that one of your colleagues, who is not involved in this process, is
reading the applicants’ files. Which one of the following is the most appropriate next step in this
situation?

o Inform the medical board.

B Report the issue to the hospital administrator.

C Report the issue to AHPRA.

D Talk to your colleague.

E Report to your supervisor.


Option D is correct

Anyone can make a voluntary notification ( raise a concern), but under the National Law only registered health
practitioners, employers, and health education providers are required to make a mandatory notification.

Separate guidelines have been developed for mandatory notification about registered health practitioners and
registered students. These guidelines apply to both (1) registered health practitioners and (2) employers of
registered health practitioners (even if they are not registered practitioners themselves).

There are four concerns that may trigger a mandatory notification. Depending on the type of concern, you must
assess the risk of harm to the public when deciding whether to make a mandatory notification.

These four concerns are:

• Impairment of the practitioner


• Intoxication while practising
• Significant departure from accepted professional standards
• Sexual misconduct.
There are different thresholds that trigger a mandatory notification depending on whether you are making a
notification as a treating practitioner, non-treating practitioner, employer, or education provider.

A treating practitioner is a practitioner who becomes aware of the concern while providing treatment to another
practitioner. The threshold for making a mandatory notification as a treating practitioner is higher than for other
notifier groups. This is to give practitioners the confidence to seek help without the fear of a mandatory
notification. The threshold for treating practitioners to make a mandatory notification about impairment,
intoxication while practising, and significant departure from accepted professional standards is when there is a
substantial risk of harm to the public.

Before making a mandatory notification, a notifier must form a reasonable belief that the incident or behavior that
led to a concern actually occurred and that a risk to the public exists. As this practitioner has not revealed the
information in the applicants’ files to anyone yet, the most appropriate step is to talk to him first and advising
against his conduct.

If you form a belief on reasonable grounds that he would continue this misconduct, mandatory reporting to
medical board via AHPRA ( options A and C) or to the administration or your supervisor ( options B and E)
depending on the workplace policies can be considered next.
Question ID 1592

Title Management of an aggressive Aboriginal boy in the Emergency Department

Notes
Q Flag as important

Font Size: A A A

An 18-year -old Aboriginal man is brought to the Emergency Department of a local hospital. He is
very agitated, hostile, and aggressive. When approached for care, he becomes even more
distressed and starts yelling at the staff and threatening to kill them if they touch him. Verbal de-
escalation fails to calm him down. After calling the security staff to take over the situation, which
one of the following would be the most appropriate next step in management?

o Discharge him.

B Call the police.

C Involuntary admission to the hospital.

D Asking for an Aboriginal health worker.

E Restrain him.
An 18-year-old Aboriginal man is brought to the Emergency Department of a local hospital. He is
very agitated, hostile , and aggressive. When approached for care, he becomes even more
distressed and starts yelling at the staff and threatening to kill them if they touch him. Verbal de-
escalation fails to calm him down. After calling the security staff to take over the situation, which
one of the following would be the most appropriate next step in management?

X QA Discharge him.

X QB Call the police.

X Qc Involuntary admission to the hospital.

y D Asking for an Aboriginal health worker.

Restrain him.
*O

Option D is correct

According to risk management protocol in Australia, calling the security is the most appropriate step if verbal de-
escalating fails. However, the situation with Aboriginals and Torres Strait Islanders is different. Aboriginal hospital
experience can be very stressful for Aboriginals and also bewildering for health workers.

In such cases, Aboriginal and Torres Strait islander hospital liaison officers (IHLO’s) play an important role in
making rapport and building relationship with Aboriginal patients. Therefore, it is strongly recommended that prior
to any physical restraining or contacting the police, the IHLO or Aboriginal health worker is contacted to be
present and de-escalate the situation.

The main reason for such necessity is the cultural differences and the familiarity of Aboriginal health workers with
the background and sensitivities of this group. For example, while a caring eye contact is usually perceived as
empathy from an aggressive patient, it is unacceptable and provoking for Aboriginal Australians.

The security staff also must be present in case they are needed but no intervention should take place as long as
possible, and until the Aboriginal health worker takes over.

Other options may be considered after appropriate intervention by and advice from an IHLO or Aboriginal health
worker.
Question ID 1532

Title Reposnse to request of test results of a 16-year-old girl by her mother

Notes
Q Flag as important

Font Size: A A A

A mother brings her 16-year-old daughter for assessment of her knee pain. In the examination
room and while you are examining the girl, she privately tells you that she has had unprotected sex
and asks for Chlamydia testing. You take a swab for testing and send it to the laboratory, the result
of which comes back positive. Few days later, the mother calls you and says that her daughter has
told her all about it and the test you performed and wants to know the test results. Which one of the
following is the most appropriate action you should take ?

o Tell the mother about the test result.

B Tell her: “I cannot tell you anything about your daughter ’s test results. No STD test result
can be discussed or revealed over the phone, not even to the patient.”

C Tell her: “please ask your daughter to call for the results.”

D Tell her: “please bring your daughter for another appointment.”

E Tell her: “you should come over for test results because the issue cannot be discussed
over the phone.”
Font Size: A A A

A mother brings her 16-year-old daughter for assessment of her knee pain. In the examination
room and while you are examining the girl, she privately tells you that she has had unprotected sex
and asks for Chlamydia testing. You take a swab for testing and send it to the laboratory, the result
of which comes back positive. Few days later, the mother calls you and says that her daughter has
told her all about it and the test you performed and wants to know the test results. Which one of the
following is the most appropriate action you should take ?

x o Tell the mother about the test result.

x QB Tell her: “I cannot tell you anything about your daughter ’s test results. No STD test result
can be discussed or revealed over the phone, not even to the patient.”

y Q Tell her: “please ask your daughter to call for the results.”

x QD Tell her: “please bring your daughter for another appointment.”

Tell her: “you should come over for test results because the issue cannot be discussed
X O
over the phone.”
19:02 Fri 24 Apr v <§> 54 % i
Although the age of maturity is 18 years old according to the Common Law, younger individuals can still consent
to their own medical and dental treatments if they adequately understand what is proposed as treatment, its
consequences and the material risks associated with it. This is referred to as ‘Gillick competence rule.’ The term
Gillick-competent or mature minor is used for such minors. As a rule, minors mature enough to consent are
medicolegally owed the same duty of confidentiality as adults. Confidentiality becomes even more important in
this age group because major health problems which can occur in adolescence include sexually transmissible
diseases, unwanted pregnancies, substance misuse and mental disorders. These are delicate and sensitive
issues for which early intervention is desirable. However, adolescents’ concerns about confidentiality can be a
barrier to their accessing to health services. When adolescents understand a service is confidential, they are
more likely to disclose information about behaviors that may put or have put their health at risk, to seek health
care, and to return for follow-up.

Understandably, parents have an interest in being informed of and knowing about their children’s health
problems. However, disclosure of such information without the adolescents’ consent will have negative effect on
therapeutic relationship between the treating doctor and the patient. Most studies show that a considerable
number of adolescents did not visit their healthcare providers - despite wanting to do so - because they were
worried that their parents would find out.

In approach to such health issues in adolescents, the most appropriate approach is encouraging the young
person to involve at least one parent as an understanding parent is an invaluable source of support and relief;
however, if the minor refuses, confidentiality should be respected and not breached.

NOTE - There are exceptions to confidentiality even if the minor has not consented to . These exceptions may
arise when there is a serious and imminent threat to the life or health of the individual (e.g. suicide) or another
person (e.g., homicide or transmission of serious infectious diseases.

NOTE - In practice, doctors should always encourage adolescents to inform their parents, particularly for
complex or complicated issues because parents are generally best source of support for their children.
However, if, despite encouragement, an adolescent refuses to inform his or her parents, confidential health
care can be provided as long as the doctor is satisfied that the adolescent is a mature minor and that the
treatment offered is in the adolescent’s best interests. In profound or life-altering procedures, such as
sterilization (in a person with intellectual disability) or gender reassignment, not even parents can consent on
behalf of their child; the Family Court must decide.
This girl requested Chlamydia testing in private, implying that she wanted the issue to be confidential. However,
even if the mother was present and aware of the test , disclosure of the test result to anybody other than the girl
without her expressed consent would be inappropriate and should be avoided; therefore, the most appropriate
response to the mother ’s request is that you are not allowed to tell anybody but her daughter about the results
and she should personally call or come for that. She can ask the test results from her daughter after they are
discussed.

(Option A ) Telling the mother about her daughter ’s test result is a breach of confidentiality and not appropriate.

(Option B ) Although you should not tell the mother about the test result, this is due to confidentiality issues, and
not because an STD test result cannot be discussed over the phone .

(Option D ) The decision as to whether the girl’s mother accompanies her in the next appointment should be
made by the girl. The next visit is very likely to include discussion about treatment option, safe sex education and
arrangement for follow-up visits. These cannot be productively and efficiently discussed in the presence of the
mother unless the girl willingly consents to.

(Option E ) You cannot tell the mother about the test results over the phone, in person or in any other way, unless
the girl has consented to disclosure.
Question ID 1526

Title Management of a situation where an HIV- positive patient refuses to inform his partner

Notes
Q Flag as important

Font Size: A A A

You are counselling a 42-year-old patient of yours who was diagnosed with HIV infection last
month. He has been in a stable relationship with his girlfriend for the past eight months. He has
had three visits to your office for consultation regarding referral and antiretroviral therapy as well as
further investigations for other sexually transmissible infections ( STIs). He has no STI other than
HIV. In the past visits, you talked about the importance of contact tracing. You told him that he
should talk to her girlfriend about his disease and brings her in for HIV testing but every time he
adamantly insists that it is his secret and he does not want anyone to know about it , her in
particular. In this visit , her girlfriend is accompanying him and is sitting in the waiting room. You
again emphasize that he should inform her because otherwise her health is at more risk. He again
refuses to tell her. Which one of the following is the most appropriate action you should take ?

o Respect the patient's wishes.

B Obtain a court order to inform his partner.

C Consult with the ethics committee.

Personally notify her or notify the health department.

E Advise safe sex.


You are counselling a 42-year-old patient of yours who was diagnosed with HIV infection last
month. He has been in a stable relationship with his girlfriend for the past eight months. He has
had three visits to your office for consultation regarding referral and antiretroviral therapy as well as
further investigations for other sexually transmissible infections ( STIs). He has no STI other than
HIV. In the past visits, you talked about the importance of contact tracing. You told him that he
should talk to her girlfriend about his disease and brings her in for HIV testing but every time he
adamantly insists that it is his secret and he does not want anyone to know about it , her in
particular. In this visit , her girlfriend is accompanying him and is sitting in the waiting room. You
again emphasize that he should inform her because otherwise her health is at more risk. He again
refuses to tell her. Which one of the following is the most appropriate action you should take ?

Respect the patient's wishes.


X O

x QB Obtain a court order to inform his partner.

Qc Consult with the ethics committee.

Personally notify her or notify the health department.


X OD

X o Advise safe sex.


HIV is a sexually transmissible infection (STI). Contact tracing of sexual partners has a crucial role in
management of sexually transmissible infections (STIs). Initiation of contact tracing is the responsibility of the
diagnosing clinician.

Contact tracing starts with a conversation with the index patient about informing their partners. The patient can
decide to inform their own contacts ( patient referral) or organize for someone else to inform them (provider
referral). Patient referral is the most common type of contact tracing used in general practice. For this type of
contact tracing to be successful, it is important that the diagnosing doctor informs the patient about who needs to
be informed and what information needs to be given. If the patient decides to use provider referral, the
diagnosing doctor can collect the contact person's details and either notify the contacts or pass the details to a
practice nurse or a sexual health clinic who can undertake this.

Both methods can be anonymous or not, and both can be performed using a range of methods including in
person, telephone, SMS, email or letter. Either method is acceptable. While patients are usually willing to inform
regular partners, for casual contacts or ex-partners often other non-direct methods are preferred by the patients.

However, due to significance of HIV, provider referral is the preferred method of contact tracing. Contact tracing
starts with recent sexual or needle-sharing partners. Outer limit is the onset of the risky behavior or the last
known negative HIV test result. If the index patient has donated or received blood products, semen, or body
tissue, the relevant authority should be contacted as well. For those whose HIV has been detected due to a
recent TB diagnosis, contact tracing for TB is also important.

At some occasions, however, the patient refuses to use either patient or provider referral to inform his/her
contacts despite full discussion and explanations. In such situations, it is the responsibility of the diagnosing
doctor to start contact tracing even without the patient’s consent. The method to consider first is notification of the
authorized body in that state. Such authorities either take the matter in hand directly, or delegate the matter to the
diagnosing physician, a nurse practitioner, ora suitably qualified and experienced counselor.

One important point to consider is that direct notification of the index patient’s sexual contacts should be
performed after approval by the ethics committee and after counselling with medical indemnity. Once approved,
the patient should be provided with a written advice that the patient must notify the partner, and if the patient still
refuses to do so then the doctor has the right to inform his/her sexual contacts.

Of the given options, consulting with the ethics committee for a legally-supported means of contact tracing is the
most appropriate one; however, notifying the health authorities would be the best action if it was among the
options.

(Option A ) The patient wishes should be respected unless they put others at risk . If the patient ignores
instructions about informing his/her sexual contacts, his wishes cannot be followed because you have a duty of
care for his sexual contacts as well and must notify the relevant authorities.

(Option B) Contact tracing for a notifiable disease against the patient’s will does not need a court order. The
diagnosing physician can notify relevant authorities if, after complete explanation of the importance of contact
tracing and its different methods, the patient refuses to inform contacts.

(Option D) While notifying the health authorities is the most appropriate option, direct notification of contacts of
this patient without his consent is not an appropriate option unless permitted and approved by health authorities
and after careful consultation with ethics committee and/or medical indemnity. Even so, the patient should be
provided with a written advice that you will notify his contacts personally if he fails to do so.

(Option E) Safe sex practice is an appropriate advice to give every patient with or without an STI but does not
eliminate the need for contact tracing and mandatory notification.
Question ID 1520

Title Action plan for an HIV- positive patient who is refusing contact tracing

Notes
Q Flag as important

Font Size: A A A

Glen is a 32-years old patient of yours who has been recently found out to be HIV positive. Maggie,
his wife, is also a patient in your office. In the last visit and after telling Glen about his diagnosis
and counseling about treatment options and preventive measures, you advised that he should tell
her wife about his diagnosis and that she needs HIV testing, and if positive, treatment. Despite full
explanations about the risks to his wife and the methods to inform her, he still refuses to tell his
wife because he believes this could ruin their marriage and that he cannot afford losing her in such
situation. You again explain to him that it is necessary for her good. He still refuses. Which one of
the following would be the most appropriate next step in management?

o Inform health authorities.

B Inform his wife.

C Respect his confidentiality.

0 Tell him that she should practice safe sex as of now.

E Apply for a court order for notification.


o Inform health authorities.

x QB Inform his wife.

x Qc Respect his confidentiality.

x QD Tell him that she should practice safe sex as of now.

X OE Apply for a court order for notification.

Option A is correct

Contact tracing of sexual partners has a crucial role in management of sexually transmissible infections (STIs ).
Initiation of contact tracing is the responsibility of the diagnosing clinician.

The objectives of contact tracing include the following:

• To prevent re-infection of the index case


• To treat the possibly infected contacts and minimize complications in them
• To reduce the population prevalence of STIs in the community
Contact tracing starts with a conversation with the index patient about informing their partners. The patient can
decide to inform their own contacts ( patient referral) or organize for someone else to inform them (provider
referral). Patient referral is the most common type of contact tracing used in general practice. For this type of
contact tracing to be successful, it is important that the diagnosing doctor informs the patient about who needs to
be informed and what information needs to be given. If the patient decides to use provider referral, the
diagnosing doctor can collect the contact person's details and either notify the contacts themselves or pass the
details to a practice nurse or a sexual health clinic who can undertake this.

Both methods can be anonymous or not, and both can be performed using a range of methods including in
person, telephone, SMS, email or letter. Either method is acceptable . While patients are usually willing to inform
regular partners, for casual contacts or ex-partners often other non-direct methods are elected.
Both methods can be anonymous or not, and both can be performed using a range of methods including in
person, telephone, SMS, email or letter. Either method is acceptable. While patients are usually willing to inform
regular partners, for casual contacts or ex-partners often other non-direct methods are elected.

However, due to significance of HIV, provider referral is the preferred method for contact tracing in a patient with
HIV infections. Contact tracing starts with recent sexual or needle -sharing partners . Outer limit is the onset of the
risky behavior or last known negative HIV test result. If the index patient has donated or received blood products,
semen , or body tissue, the relevant authority should be contacted as well. For those whose HIV has been
detected due to a recent TB diagnosis, contact tracing for TB contacts is also important.

If, despite adequate consultation about the importance and means of informing sexual contacts, the index case
refuses to use either patient referral or provider referral to inform his/her contact, it is the responsibility of the
diagnosing doctor to start contact tracing without the patient’s consent. The method to consider first is notification
of the authorized body in that state. Such authorities either take the matter in hand, or delegate the matter to the
diagnosing doctor, a nurse practitioner, or a suitably qualified and experienced counselor.

Depending on the state such authorized bodies are :

• Australian Capital Territory (ACT): Chief Health Officer


• New South Wales (NSW): Director General of NSW Health
• Queensland (QSL): Chief Executive of QLD Health
• South Australia (SA): Clinic 257 Contact Tracing Officer - the doctor can notify directly or may refer the
patient
• Victoria: Partner Notification Officer
• Western Australia ( WA ): regional Public Health Units

NOTE - The Tasmanian Act states that a medical practitioner who becomes aware that his or her HIV-positive
patient has not taken all reasonable measures and precautions to prevent the transmission of HIV may, after
consultation with an approved specialist medical practitioner, inform any sexual contact of that HIV-positive
person, and will not be liable to any civil or criminal liability in relation to that action.

This patient has refused to inform his wife despite being consulted about the importance of contact tracing and
the means it can be done but still ignores your instructions. In such circumstance, the most appropriate action
would be informing the wife through notifying an authorized body defined by the health department.

(Option B ) Informing the patient' s wife directly is not an appropriate option unless such responsibility is
delegated by the authorized body in the health department, or after consultation with institutional ethics
committee and discussion with the medical defence organization before disclosure. If direct notification is
approved after the above processes, the doctor should provide the patient with written advice that he/she must
notify the partners because if he/ she refuses the doctor will do that.

(Option C ) Refusing to inform sexual partners is a decision that puts others at risk and forms one of the
exceptions to patient-doctor confidentiality. By the law, the diagnosing physician has a duty of care for the
patient ’s sexual partners and has to notify the relevant authorities if the patient refuses to do so.

(Option D ) Safe sex practice is an appropriate advice to give every patient with or without an STI but does not
eliminate the need for contact tracing and mandatory notification.

(Option E) Contact tracing for a notifiable disease against the patient' s will does not need a court order. The
diagnosing physician can notify relevant authorities if, after complete explanation of the importance of contact
tracing and its different methods, the patient refuses to inform contacts.
Question ID 1518

Title Response to a mother 's request for her daughter ' s test results

Notes
Q Flag as important

Font Size: A A A

A 14-year- old girl presents to you, accompanied by her mother, for a health check. She asks to be
examined alone without her mother in the room. During the examination, she requests a Chlamydia
test. Based on your assessments, she appears to fully understand the test and the impact of the
results. You take a swab to send to a local laboratory. After one week you receive the test result
which is negative. Today, the mother has called your office, and tells you that she knows about her
daughter being tested for Chlamydia and she wants to know about the result. Which one of the
following would be the most appropriate response to her request?

o Do not tell her.

B Tell her that the test result is negative.

C Tell her that the results have not come through yet.

D Ask her to come with her daughter for the next appointment.

E Tell her that you can disclose the results only to her daughter.
A 14-year- old girl presents to you, accompanied by her mother, for a health check. She asks to be
examined alone without her mother in the room. During the examination, she requests a Chlamydia
test. Based on your assessments, she appears to fully understand the test and the impact of the
results. You take a swab to send to a local laboratory. After one week you receive the test result
which is negative. Today, the mother has called your office, and tells you that she knows about her
daughter being tested for Chlamydia and she wants to know about the result. Which one of the
following would be the most appropriate response to her request?

*o
Do not tell her.

x QB Tell her that the test result is negative .

X c Tell her that the results have not come through yet.

X Ask her to come with her daughter for the next appointment.

Tell her that you can disclose the results only to her daughter.
X O
According to Common Law, there is a presumption that minors (people under 18 ) lack the competence to consent
to medical treatment ; however, if the minor can show they have competence, the law will treat them as having the
right to consent to treatment. In other words, a person aged 18 or older is competent until proven otherwise, while
a person under 18 years is defined incompetent unless the treating physician, through assessment , forms a belief
that the minor has sufficient understanding and intelligence to enable him or her to understand fully what is
proposed. This test is sometimes referred to as the Gillick competence test, and the minor is referred to as
Gillick-competent minor or mature minor.

There is another important issue in the scenario that should be considered as well. Sexually transmissible
diseases, unwanted pregnancies, prenatal care, substance misuse and mental disorders are major health
problems which can occur in the adolescence. These are delicate and sensitive issues for which early
intervention is desirable. However, adolescents’ concerns about confidentiality can be a barrier to their accessing
health services. When adolescents understand a service is confidential, they are more likely to disclose
information about behaviors that may put or have put their health at risk, to seek health care, and to return for
follow-up. For termination of pregnancy in minors, however, additional specific rules apply.

This girl has concerns about Chlamydia (an STD) and asked you to be tested in private and without her mother
knowing of that. Since she has not given you the explicit consent to inform her mother of the issue, giving any
information in this regard to the mother would be a breach of confidentiality and inappropriate. This could ruin the
trust the girl has put in you and make her reluctant to ever come back for treatment and consultation should she
have any other health issues. Considering this fact, the most appropriate response to the mother ’s request is to
mention her daughter ’s right of confidentiality and the fact that you can disclose the results only to her daughter.
The mother knows about the test and it is likely that the daughter has told her about it. Nonetheless, this should
not lead to disclosure of the test results just based on this probability.

More importantly, once a minor is considered Gillick-competent or mature , he/she is entitled to the same
confidentiality of medical information as an adult patient.

(Option A ) Although by not telling the mother confidentiality is preserved, this option is not appropriate because
not only you should not tell her, but you should explain why you cannot tell her. You need to come clear with her
about the confidentiality issue and that is her daughter ’s decision as to whether informing her of the test results.

(Options B ) Once a mature minor is considered mature, as in this case, he/she entitles confidentiality, and giving
any information regarding diagnosis, treatment or prognosis to anyone but the patient is a breach of this
confidentiality and inappropriate.

(Option C ) This option includes giving false statements. Lying is unprofessional and inappropriate.

(Option E) Asking the mother to come with her daughter for the next appointment is not appropriate. This is a
decision the daughter should make. The girl might want to come alone for discussion in a more private visit while
she is not concerned about her mother ’s presence.
Question ID 1498

Title Response to a request for a prescription by an intern

Notes
Q Flag as important

Font Size: A A A

You are a hospital medical officer (HMO ) in a public hospitals. An intern in your ward approaches
you and says he has just started his rotation in this ward and has been under lots of stress lately.
He cannot get enough sleep and feels down and irritated most of the time. He asks you for a
prescription of some benzodiazepine to help him out. Which one of the following should be the
most appropriate response at this situation?

Report him to the coordinator of intern program.


OA

B Report him to the medical board.

C Ask him to see his general practitioner ( GP ).

0 Prescribe benzodiazepines for him.

E Refer him to a support group.


19: C£ Fri 24 Apr Report him to the coordinator of intern program. ® 54 % (fo

x QB Report him to the medical board.

C Ask him to see his general practitioner ( GP ).

X D Prescribe benzodiazepines for him.

Refer him to a support group.


X O

Option C is correct

Most interns find their intern year enjoyable and satisfying but it will also be intellectually, physically and
emotionally challenging at times. It is advised that interns speak to others around them who have been or are
going through similar experiences for the best advice.

When a doctor, including interns and medical students, feels unwell either physically or emotionally, the best
advice to give is seeking professional help. Like everybody else, healthcare professionals should have a regular
general practitioner (GP ) as their first point of contact with the healthcare system. This holds true about this
situation as well. You should encourage this troubled intern to see his GP, or if he does not have any, it is about
time he had one .

The Medical Board , as a part of code of conduct and good medical practice for medical practitioners
recommends against self-assessment when it comes to the doctor ’s own medical care and advocates consulting
with an independent doctor for professional advice.

(Option A ) Reporting to the coordinator of interns program is the option when you form a reasonable belief that
the intern’s condition is jeopardizing the patients’ care and health, which is not the case here. He, with good
intentions, has sought help for his problem, and you have not witnessed or been informed of him putting patients
at risk , or deviating from professional manner.

(Option B ) Although interns hold a provisional registration with the Australian Health Practitioner Regulation
Agency (AHPRA ), notifications should be made to the manager or coordinator of interns program in the hospital
they are working, and not directly to the AHPRA and the medical board.

(Option D ) This intern should be advised to go through the standard path of healthcare as for every member of
the community. Any prescription of therapy for him should only be done after a full evaluation of different aspects
of his problem. Therefore, prescription of therapies of any kind, prior to this assessment is inappropriate and an
incorrect option.

(Option E) Although referral to a support group seems a benign and harmless option in this situation, it is better
to be left to the intern ’s GP for discussion, and offered by someone who is fully in charge of his medical care.
Question ID 1482

Title The period during which a referral is valid

Notes
Q Flag as important

Font Size: A A A

Jane is in your office for her laboratory test results you ordered 15 days ago when you suspected
she may have systemic lupus erythematosus (SLE) based on her history and clinical findings. The
test results confirm your diagnosis. You write a referral letter to a local rheumatologist for
consultation. She wants to know for how long this referral letter is valid. Which one of the following
would be you correct response to her question?

o Three months from the date of issue.

B Three months from the first visit by the specialist.

C Six months from the first visit by the specialist.

D One year from the date of issue.

E One year from the first visit by the specialist.


Jane is in your office for her laboratory test results you ordered 15 days ago when you suspected
she may have systemic lupus erythematosus (SLE) based on her history and clinical findings. The
test results confirm your diagnosis. You write a referral letter to a local rheumatologist for
consultation. She wants to know for how long this referral letter is valid. Which one of the following
would be you correct response to her question?

xO Three months from the date of issue.

x QB Three months from the first visit by the specialist.

x Oc Six months from the first visit by the specialist.

X QD One year from the date of issue.

QE One year from the first visit by the specialist.

Option E is correct

If supplied by a general practitioner (GP), the law states that a referral is valid for a single course of treatment for
a period of 12 months (one year) after the first service given in accordance with the referral. Despite this
limitation, the referring practitioner can specify an alternative time period such as three, six , or 18 months or
indefinite. Indefinite referrals are not usually made because patients often have a specific condition requiring a
certain amount of time and treatment from specialists.

Note that the referral period begins on the date of the first specialist visit , not on the date the referral was written.
This is frequently misunderstood, including by general practitioners, specialists and their receptionists.

Other rules to remember are:

• Referrals from a specialist are valid for only three months (after the first visit by the practice the patient has
been referred to).
• A referral for admitted patients is valid for three months, or the duration of the admission, whichever is the
longer.
Question ID 1480

Title Responding to the request of signing a patient ' s will from his spouse

Notes
Q Flag as important

Font Size: A A A

You are a general practitioner in a general clinic. Albert and Mandy, husband and wife, are your
regular patients. Albert has diabetes and recently survived a myocardial infarction. Mandy has
come to you today, and shows you a will that she says is signed by Albert. She says that Albert has
made changes to his previous will and now this his new will signed by him. She asks you to sign
the will as witness. What should be your response to her request?

Sign the will.


OA

B Refuse to sign the will .

C Call Albert and ask if he confirms the will.

D Seek legal advice from your medical indemnity.

E Ask you clinic manager if you can sign the will.


You are a general practitioner in a general clinic. Albert and Mandy, husband and wife, are your
regular patients. Albert has diabetes and recently survived a myocardial infarction. Mandy has
come to you today, and shows you a will that she says is signed by Albert . She says that Albert has
made changes to his previous will and now this his new will signed by him. She asks you to sign
the will as witness. What should be your response to her request?

X A Sign the will.

B Refuse to sign the will.

x Qc Call Albert and ask if he confirms the will.

Seek legal advice from your medical indemnity.


X O

Ask you clinic manager if you can sign the will.


X O
Option B is correct

Under succession law, a will must be signed in front of at least two people as witnesses. This is required by the
legal formalities for making a valid will. The purpose of witnesses for a will is provision of a safeguard for
prevention of fraud and forgery.

Any person above the age of 18 years with mental capacity and credibility to give evidence in a court of law can
witness a will. The only exceptions are those who are unable to see (for example are visually impaired) to see the
act of signing and those who will inherit under the will. The latter however has been changed is some states. For
example, in Victoria , someone who will benefit under the will can sign as a witness.

The process of witnessing a will is as follows:

• First, the will-maker must sign the will in front of two or more witnesses, all present at the same time and in
the same place.
• Witness must be mentally competent and be able to see the will-maker make their signature.
• At least two witnesses having attested the will then sign their names in confirmation that the will-maker ’s
signature, made in their presence, was genuine.

For you, the most appropriate response to Mandy ’s request must be refusal to sign the will, because you have
not witnessed Albert’s signing the will in person. You must tell Mandy that you cannot witness the will because
the signature was not made in your presence and you cannot witness something you have not witnessed.
Moreover, you should tell Mandy that only your signature as a witness cannot validate the will because at least
two witnesses must be present at the same time and in the same place for a valid witnessing process to the will.

Calling Albert to ask if he confirms the will ( option C ) is incorrect too. Even if he says that: “it is OK doc. This is
my signature and you can go ahead” you cannot still witness the will because you have not seen him in person
signing the will .

Signing the will (option A) is the most inappropriate action to take. Signing as a witness what you have not
witness is ethically and professionally wrong.

It is better to be familiar with some basic legal concepts that may arise in your daily practice. In this case, the
issue is quite straight forward and based on common sense. As an ethical basic you cannot witness something
you have not personally witnessed . The case is so clear that no further action including seeking legal advice from
your medical indemnity (option D) , or asking your clinic manager if you can sign the will ( option E ) is required.
Calling your medical indemnity for this issue is like you call them to ask if you can tell a lie .

However, in cases where you are unsure about what to do, seeking legal advice from available reliable resources
such as your medical indemnity sounds reasonable and wise.
Question ID 1389

Title Action plan in mangement of inappropriate medication dosing by an intern

Notes
Q Flag as important

Font Size: A A A

During your shifts, you come to know that one of the interns in your ward has been giving
inappropriate doses of medications to patients at different occasions for the past several weeks.
Which one of the following is the best action to take ?

o Talk to the intern about your concern and his mistake.

B Report to the supervisor of intern training.

C Tell the ward nurse to help the intern.

D Report to Medical Board.

E Report to the hospital manager.


During your shifts, you come to know that one of the interns in your ward has been giving
inappropriate doses of medications to patients at different occasions for the past several weeks.
Which one of the following is the best action to take?

X A Talk to the intern about your concern and his mistake.

QB Report to the supervisor of intern training.

X c Tell the ward nurse to help the intern.

x o Report to Medical Board.

Report to the hospital manager.


X O

Option B is correct

Issues related to interns in a hospital or clinic, whether it is impairment, misconduct , or mistakes are often directly
reported to the supervisor of intern training ( SIT) or director of clinical training. One exception is where there is
immediate and serious risk to the wellbeing or safety of patients or other staff, in which case action should be
taken immediately. For example, if you witness that an intern is about to administer an inappropriate dose of the
drug you should tell him/her about it before he/she can put more risk to the patient’s health.

NOTE - It is the responsibility of the SIT or the director of clinical training to notify the relevant authorities such as
AHPRA in case of any notifiable misconduct or impairment.
Question ID 1341

Title Approach to misconduct of a senior surgeon by an intern

Notes
Q Flag as important

Font Size: A A A

You are an intern in the surgery ward. There is an attending surgeon, who has had several anger
outburst since you started the ward. Once he becomes angry, he shouts at ward nurses and other
staff. At one occasion, he confronted an operating room nurse during a surgery due to what he
believed was irresponsibility from the nurse. When she tried to explain to him, he became angry
and left the operating room after he shouted and insulted the nurse. Which one of the following is
the best action you should take ?

o Talk to the surgeon directly.

B Talk to one of the attending surgeons.

C Raise the problem at the interns meeting.

D Report to the director of clinical training.

E Report to the hospital manager.


You are an intern in the surgery ward. There is an attending surgeon, who has had several anger
outburst since you started the ward. Once he becomes angry, he shouts at ward nurses and other
staff. At one occasion, he confronted an operating room nurse during a surgery due to what he
believed was irresponsibility from the nurse. When she tried to explain to him, he became angry
and left the operating room after he shouted and insulted the nurse. Which one of the following is
the best action you should take ?

X A Talk to the surgeon directly.

X QB Talk to one of the attending surgeons.

X Qc Raise the problem at the interns meeting.

o Report to the director of clinical training.

Report to the hospital manager.


*O

Option D is correct

In case an intern has any concerns regarding any supervision, lack of guidelines, problems with education
and training, observation of misconduct by other interns, registrars, consultants, and other colleagues, or
any conflict in workplace, the problem should be reported directly to the supervisor of intern training or the
director of clinical training and advice is sought from him / her. These persons can also play a pastoral role in
such events in giving advice regarding the situation or reporting the matter if deemed necessary. An
supervisor of intern training or a director of clinical training is often the first point of contact for interns.
Talking to the surgeon directly (option A), or to one of attending surgeons ( option B), or reporting to
hospital manager (option E) are decisions that should be left to or at least made after consultation with the
supervisor of intern training or the director of clinical training.

Raising the issue at the intern meeting ( option C) leads to unnecessary publicizing of the issue that could
be addressed in a more personal and confidential manner.
Question ID 1333

Title Approching a physician with possible drug intoxication

Notes
Q Flag as important

Font Size: A A A

After a night shift, an emergency department physician goes to a public bar where he meets
his friend. There, they use amphetamines for pleasure and spend some hours together in the
bar drinking alcohol. You, as another doctor working in the same hospital, see him using
drugs. His next shit starts in two hours. Which one of the following is the most appropriate
next action to take?

o Inform the Medical Board.

B Inform the Australian Health Practitioner Regulation Agency (AHPRA).

C Leave him and say nothing.

Talk to him about not going to work under drug influence.

E Report to the director of the emergency department.


x QB Inform the Australian Health Practitioner Regulation Agency (AHPRA).

X Qc Leave him and say nothing.

Talk to him about not going to work under drug influence.


O

X QE Report to the director of the emergency department.

Option D is correct

According to section 140 of National Law, ‘notifiable conduct’ occurs when a medical practitioner has:

• practiced the practitioner ’s profession while intoxicated by alcohol or drugs; or


• engaged in sexual misconduct in connection with the practice of the practitioner’s profession; or
• placed the public at risk of substantial harm in the practitioner 's practice of the profession because the
practitioner has an impairment; or
• placed the public at risk of harm because the practitioner has practiced the profession in a way that
constitutes a significant departure from accepted professional standards.

Although this doctor has used drugs and alcohol, he is not practicing under influence and has used illicit drugs
and alcohol outside the workplace and in his private life. You have not seen him practicing while intoxicated,
neither have you formed a reasonable belief that he is doing so. Unless he practices under influence, mandatory
reporting to AHPRA (option B) or medical board (option A ) is not necessary.

On the other hand, you know that he is attending the emergency department in 2 hours and might be still under
influence when he starts working; thus putting his patients at risk. Now, this is your duty to prevent harm to
patients. For this reason, leaving him and saying nothing ( option C ) is not an appropriate option as well.

In this case, talking to him about not going to work is the most appropriate next thing to do. If he seems so under
influence to not understand the situation and you form a belief that he may practice while intoxicated , notifying
the director of the emergency department of the situation ( option E) would be a wise decision.

TOPIC REVIEW

Approaching a colleague who is practicing under alcohol or drug influence:

STEP 1 - Duty of care to patient(s) (patients’ safety) - talk to the colleague and send him home. If difficult seek
help from another colleague or the clinical director.

STEP 2 - Duty of care to colleague - make sure that your colleague is safe and check to ensure he/ she has
reached home safely. Suggest occupational health referral and insist that his/her behavior was not appropriate.

STEP 3 - Duty of care to your workplace ( hospital/clinic) - keep accurate record of the incident and inform the
clinical director. Consider notifying AHPRA if required in cases of obligatory notifications.
Question ID 1229

Title The most important initial action to take after disclosure of domestic violence

Notes
Q Flag as important

Font Size: A A A

A 32-year- old woman presents to your GP practice with complaints of disturbed sleep, colicky
abdominal pain and headaches. During examination, you notice several bruises over her arms,
legs and chest. Upon further inquiry, she confides in you that she has been beaten by her husband
and this happens almost every few days. She then breaks in tears and tells you that she feels
helpless and hopeless and is living in constant fear and stress. She insists that you not tell
anybody about this, especially the police. Which one of the following is the most appropriate next
step in management?

o Take photos of the bruises.

B Call the police.

C Tell her that she should call the police.

Give her the contact numbers for a safe shelter.

E Counsel her about domestic violence.


19:08 Fri 24 Apr ^® 55% m
X c Tell her that she should call the police .

X D Give her the contact numbers for a safe shelter.

E Counsel her about domestic violence .

Option E is correct

Family violence is a coercive and controlling behavior by a family member that causes physical, sexual and/or
emotional damage to others in the family, including causing them to live in fear and threatening to harm people
pets or property. Family violence is most perpetrated by one partner towards another, or by an adult towards a
child. Other forms include elder abuse or sibling abuse.

Establishing trust and rapport is of significant importance when domestic violence is disclosed by a patient.
Doctors should be empathic, avoid judging the patient and be direct about the illegal nature of violence. Some
patient might believe that the bad attitude toward them is their fault. The victim of domestic violence should be
counselled properly and openly about the nature of domestic violence. They should fully understand that assault,
in any form, occurring between family members is a criminal offence, unacceptable and prosecutable, and that
they can take steps to control the situation as the most appropriate next step in management of the situation.

(Option A ) Recording the patient’s explanations, physical exam findings and, with the patient ’s consent, taking
photographs of the injuries is an important step. Such documents can be used as invaluable pieces of evidence
in a court of law if the patient decides to press charges against the perpetrator. However, she will feel more
confident and experiences mutuality if they are consulted beforehand. Although documenting the account of the
event and the injuries is important, it is better done after counselling.

(Option B ) It is important that she learns her legal options, including calling the police. However, this cannot be
done without her consent. By law, every adult person is mandated to report to the police or relevant authorities if
they are sure or highly suspected that family violence has occurred or is occurring. Healthcare professionals,
especially doctors, are exceptions to this rule because such disclosures to a healthcare professional occurs
based on a therapeutic relationship, with confidentiality as a cornerstone of this relationship. Confidentiality
cannot be breached without the patient’s informed consent unless there is an imminent serious risk to
their health, or if there are children involved and in immediate risk.

(Option C ) Victims of domestic violence have legal options including informing the police. Patients should be
made aware of such options and encouraged to use them based on their own decisions. This, however, is best if
advised after appropriate counselling.

(Option D ) If the patient is reluctant to contact authorities such as the police, family violence services, or other
relevant authorities, making a safety plan with the patient is a very important next step. A safety plan may include
making a list of emergency numbers and considering a safe place for the patient to go to and how the patient will
get there. Such issues can be brought up and discusses after appropriate counselling. She should also be
advised to leave the violent environment and stay with a family or friend, or to seek shelter in a refuge center as
soon as possible.
Question ID 1228

Title Approach to domestic violence after it is disclosed by the victim

Notes
Q Flag as important

Font Size: A A A

Martha , 54 years old, is a patient of yours, who has come to your office today for a health check.
During the session you notice that she is sad and worried, and is vaguely trying to communicate
about her concerns. Further probing reveals the issue. Her 16- year-old son is an addict and a drug
dealer. She has been frequently beaten by him when she refuses to give him money for drugs.
Which one of the following is the most important next step to consider regarding her problem?

Call the police.


O

B Tell her to seek shelter in a women’s refuge center.

C Inform the child protection service.

D Admit the son to a rehabilitation center.

E Arrange an appointment with the son .


19 '
°^/lafitia,^ years old, is a patient of yours, who has come to your office today for a health cfieCk 55 /o

During the session you notice that she is sad and worried, and is vaguely trying to communicate
about her concerns. Further probing reveals the issue. Her 16- year-old son is an addict and a drug
dealer. She has been frequently beaten by him when she refuses to give him money for drugs.
Which one of the following is the most important next step to consider regarding her problem?

X A Call the police.

B Tell her to seek shelter in a women’s refuge center.

X c Inform the child protection service.

Admit the son to a rehabilitation center.


X O

Arrange an appointment with the son.


X O

Option B is correct

Of the options, advising Martha to seek shelter in a women’s refuge center is the most appropriate one. She must
move from the violent environment to stay with a relative or friend or a women’s refuge center to reduce the risk
of further harm while further action and follow-up is planned for her.

(Option A ) Although it is important that Martha is informed of her legal options, including calling the police, this
cannot be done without Martha’s consent. Every adult person is mandated by law to report to the police or
relevant authorities if a belief is formed, based on reasonable grounds or upon disclosure by the victim that family
violence has occurred or is occurring. However, if such issue is disclosed to a healthcare professional as a part of
a therapeutic relationship, the healthcare worker often should not call the police without the patient’s consent.
Exceptions must be made when there is an imminent serious risk to one ’s health, or if there are children involved
and in immediate risk.

(Option C ) Informing child protection services in not appropriate because the one requiring protection in this
scenario is Martha not his son.

(Option D ) Admission of the boy to a rehabilitation center is not a priority compared with Martha’s safety and
wellbeing, nor can it be done without the son’s consent.

(Option E) Arranging an appointment with the son may be planned later on after discussion with Martha but does
not take precedence over a safety plan for Martha’s safety.
Question ID 1203

Title Response to false negative comments on Facebook

Notes
Q Flag as important

Font Size: A A A

One of your friends calls and tells you that he has come across some negative comments
regarding your practice as a general practitioner on Facebook. You check and realize that a Tom
has written many negative posts about you and the way you practice and approach your patients.
One of his posts is a detailed story about how you mismanaged him for a ore throat three weeks
ago. You check your files and you recognize him; a patient you treated for a viral pharyngitis and
refused to give him antibiotics despite his insist. Which one of the following is the most appropriate
action you should take in management of this situation?

o Inform your medical defense organization and seek advice.

B Give feedback on his posts by explaining the nature of his condition and why you
refused his requested treatment.

C Write to ask him to remove the post.

D Ignore him and his posts.

E Contact Facebook and use its policies for removing the post.
One of your friends calls and tells you that he has come across some negative comments
regarding your practice as a general practitioner on Facebook. You check and realize that a Tom
has written many negative posts about you and the way you practice and approach your patients.
One of his posts is a detailed story about how you mismanaged him for a ore throat three weeks
ago. You check your files and you recognize him; a patient you treated for a viral pharyngitis and
refused to give him antibiotics despite his insist. Which one of the following is the most appropriate
action you should take in management of this situation?

o Inform your medical defense organization and seek advice.

x QB Give feedback on his posts by explaining the nature of his condition and why you
refused his requested treatment.

X c Write to ask him to remove the post.

x QD Ignore him and his posts.

*o
Contact Facebook and use its policies for removing the post.
Option A is correct

Doctors have always been subject to reputational risks from negative comments or rumors. Nowadays and with
the enormous speed at which words are spread on social media, the risk is even more significant. The issue is
not only how but whether the healthcare entity or an individual physician should respond at all. Such situations
make a dilemma: the patient or his/her family has revealed his/her history or story on the social media but a
response by the doctor to his/her comments and explaining the account of events could be a violation of patient’s
privacy. On the other hand, ignoring such comments could be perceived as an unspoken agreement with the
complaints or lack of concern with the feedback. The desire to “correct the record” or give the other side of the
story when faced with the negative online posting is natural. However, doing so may lead to an allegation of
breach of confidentiality or other legal consequences.

When facing such problem, as a medical center or an individual doctor, there are options to consider:

• Ignoring by simply doing nothing.


• If the patient can be identified with certainty, contact the patient directly to discuss their concerns and see if
they will remove the post.
• Use the website (Facebook here) policies for removal of the post(s).
• Send a letter to the patient and/ or website owner and request the and/ or threaten that you will take legal
action for defamation.
• Start defamation proceedings.

Depending on the situation and the nature of the posts and /or comments or whether the patient can be identified
any of the above options can be choice; however, it is very important that you seek advice from an expert
colleague or your medical defense organization as the most appropriate initial step in the challenge.

(option B ) Giving feedback on the post means that you are discussing the patient 's private information , released
to you as his doctor based on the doctor-patient confidentiality. This is a breach of this confidentiality and very
incorrect. Unlike complaints that are made by a patient directly to an organization, many online complaints are
anonymous. You may think you know who made the complaint, but it may have been made by a relative or friend
who knows about the patient, and not the patient himself.

(Options C and E ) These options can be your choices as well; however, the decision as to how to respond is
best made after consultation with an expert colleague or medical defense organization.

(option D ) In most cases, ignoring such negative comments and/or posts are advisable if the nature of such
posts/comments are benign. However, in this case, where your reputation or that of the center you are working at
might be at risk, the best action to take can be considered after consultation with more expert resources of
advice .
Question ID 1192

Title Management of a pregnant woman after disclosure of domestic violence

Notes
Q Flag as important

Font Size: A A A

A pregnant woman has presented for an antenatal visit. During the session, she admits to being
beaten by her husband at home. She adds that her husband is an alcoholic and aggressive and
sometimes he even punches and kicks her. However, he is very careful not to leave any marks or
bruises as a proof of physical abuse. Which one of the following is the most important step in
response to this woman’s alleged situation?

o Call the police.

B Admit her to the hospital.

C Talk her into calling the police.

D Talk to her husband.

E Refer them for marital counselling.


A pregnant woman has presented for an antenatal visit. During the session, she admits to being
beaten by her husband at home. She adds that her husband is an alcoholic and aggressive and
sometimes he even punches and kicks her. However, he is very careful not to leave any marks or
bruises as a proof of physical abuse. Which one of the following is the most important step in
response to this woman’s alleged situation?

X A Call the police.

S QB Admit her to the hospital.

X c Talk her into calling the police.

X O Talk to her husband.

*o
Refer them for marital counselling.
Option B is correct

Domestic violence in pregnancy is a common, chronic, complex social problem which is present in all cultures.
For many women, pregnancy and the postpartum period exacerbates the violence and threats within their
relationship. A violent and jealous partner may form an aggressive behavior towards the pregnancy because he
is not prepared to ‘share’ her. Financial or sexual pressures may also be more compromised by the pregnancy.

An abusive partner will often target the breasts, stomach and genitals of their pregnant partner. Often the abuse
will start with the first pregnancy, and as a result the woman may avoid prenatal check-ups. Women who do not
seek antenatal care until the third trimester should raise a suspicion of domestic violence.

Abused pregnant women are twice as likely to miscarry compared to non-abused pregnant women. Other
obstetrics complications such as preterm labor and placental abruption are also more likely to occur in such
women. In addition to measures taken for non-pregnant victims of domestic violence, obstetric assessment and
interventions should also been considered , including an assessment of risk of physical harm to a pregnant
woman and her fetus from domestic violence. Most of the time, it is not safe for the pregnant woman to return
home . In such situation, an emergency admission can be made as a ‘place of safety '.

Admission secures the safety of the woman and her fetus for now, and provides a window of opportunity not only
for a full assessment of the potential injuries to her and the fetus, but also for discussing the effects of abuse on
her pregnancy and the fetus’s wellbeing as well as offering different options for help and improving her condition.

(Option A ) This woman is a competent adult. Calling the police without her consent is not appropriate unless
there is imminent risk to her health or when there is serious injuries such as lacerations, broken bone, gunshot
wounds or stab wounds. She does not have such injuries and is safe while in your office.

(Option C ) This woman should be advised to inform the police directly and report her problem because the police
may be able to provide more information about the patient’s legal options. The patient should be informed that in
many cases she will be able to activate or withdraw from criminal proceeding later. Medical practitioner should
also offer to report the incident to the appropriate authorities, including the police if the patient wants
this. However, it is important to respect their wishes and not pressure them into making any decisions. Any
options suggesting reporting to the police by the treating doctor without the patient’s consent or urging or pushing
the patient to call the police is incorrect. Options should be fully explained but decision must be left to the patient.

(Option D ) Talking to the husband is not appropriate at this stage because in most cases such intervention
escalates the situation and jeopardizes the woman’s condition.

(Option E) Referring the woman for marital counselling may imply that the violence is the woman’s fault.
However, it may be considered later in the course of treatment.
Question ID 1191

Title Consent for emergency colostomy in a 71 -year -old man with Alzheimer disease

Notes
Q Flag as important

Font Size: A A A

You are a surgeon in the emergency department. A 71-year-old man with documented history of
Alzheimer disease , who lives with his daughter, is brought to you. The patient is confused, cannot
communicate with you, and does not appear to understand the situation. He is in urgent need for
an immediate colostomy. You try to reach his daughter on the phone for consent but she is not
available. Which one of the following is the most appropriate next step in management?

o Proceed with the surgery.

B Wait until you can find the daughter and obtain consent.

C Do not perform the surgery.

Perform the surgery after signatures from two other doctors.

E Apply the case to the court for guardianship.


You are a surgeon in the emergency department. A 71-year-old man with documented history of
Alzheimer disease, who lives with his daughter, is brought to you . The patient is confused, cannot
communicate with you, and does not appear to understand the situation. He is in urgent need for
an immediate colostomy. You try to reach his daughter on the phone for consent but she is not
available. Which one of the following is the most appropriate next step in management?

y Proceed with the surgery.

x QB Wait until you can find the daughter and obtain consent.

X Qc Do not perform the surgery.

X D Perform the surgery after signatures from two other doctors.

X E Apply the case to the court for guardianship.

Option A is correct

In emergency situations, when the patient is permanently or temporarily unable to give consent, and there is no
carer or guardian or next of kin available to obtain consent from, and delaying the treatment will pose risk to
patient’s health, treatment can be provided without consent. This is based on the principle of duty of care and the
patient 's best interest, and assuming that the patient ’s or his/her carer would consent to treatment if he/ she was
present. However, if there is an advanced care directive made in the past by the patients, the instruction
contained in that must be followed.

(Option B) In emergency situations, slightest delay may be associated with increased risks to the patient’s
health; therefore, wasting time waiting for the daughter to arrive for consent would be inappropriate.

(Option C ) While this patient is in need for emergency treatment, deferring the surgery is not appropriate.

(Option D ) You are the healthcare professional responsible for the care and health of the patient and no approval
from other doctors is required.

(Option E) Application of the case to the guardianship court in instances where a patient is in need for
emergency treatment is not appropriate, as delay in treatment is against duty of care and the patient’s best
interest.
Question ID 1190

Title Appropriate response to aggressive behavior of an old woman during a home visit

Notes
Q Flag as important

Font Size: A A A

You are asked by a community service to check on an old woman who lives alone and is reported
by her neighbors. When you arrive, you knock but there is no response. After you find the front
door open , you call her by name and introduce yourself as you step in. When you enter her house ,
the first thing you notice is a messy living room and a dirty kitchen with unwashed dishes piling
upin the sink and rotting food on a plate. An old woman, who is disheveled and unkempt , walks out
of a room, comes towards you, and calls you by another name. You have just started introducing
yourself when she suddenly shouts at you to get out and leave her alone. When you try to calm her
down and explain who you are and why you are at her home, she suddenly attacks you and takes
a swing at you. Which one of the following should be the immediate response in this situation?

Leave the house immediately.


OA

B Call the police.

C Restrain her.

D Sedate her .

E Get a court order.


.y
~ -<§>
W WI U U I U/ U U W l I II I I 1 1\ v J V/ l V l U V y I W VyI I V/ U H W I I U l I V V W I I I U I *V I I V J I I Y V y U U I V I I V y U l I U lx » I V y k7 V »l IVJVJ

19:10 Fri 24 Apr 55% mj


by her neighbors. When you arrive, you knock but there is no response. After you find the front
door open , you call her by name and introduce yourself as you step in . When you enter her house ,
the first thing you notice is a messy living room and a dirty kitchen with unwashed dishes piling
upin the sink and rotting food on a plate. An old woman, who is disheveled and unkempt, walks out
of a room, comes towards you, and calls you by another name. You have just started introducing
yourself when she suddenly shouts at you to get out and leave her alone. When you try to calm her
down and explain who you are and why you are at her home, she suddenly attacks you and takes
a swing at you. Which one of the following should be the immediate response in this situation?

y A Leave the house immediately.

X B Call the police.

X c Restrain her.

Sedate her.
X O

Get a court order.


X OE

Option A is correct

The safety of healthcare staff is of paramount significance when dealing with an aggressive patient.

Both the house and the patient’s condition indicate that she is very likely to have a mental illness either caused
by a medical or psychiatric condition. Your presence at her home has made her aggressive to the extent of
attempting assault on you. In this situation, the most appropriate immediate response should be leaving the
house immediately to prevent further aggression and possible assault.

(Option B ) Calling the police for support is the next best step after your safety is ensured. In the presence of the
police, you can try verbal de-escalation or other necessary measures such as sedation.

(Options C and D ) Sedation or restraining deprives the patient of autonomy and should only be considered as
the last resort. When physical restraint is required, a coordinated team approach is essential, with roles clearly
defined and swift action taken ( restraint protocol ). While you are alone, trying to restrain or sedate an aggressive
patient may pose harm both to you and the patient.

(Option E) Getting a court order is not necessary at this stage. Court order for guardianship may be required
later for guardianship issues or consent to treatment.
Question ID 1189

Title Management of a 78 -year -old woman with memory decline who insists to travel alone

Notes
Q Flag as important

Font Size: A A A

A 78 -year - old woman, who is a patient in your GP clinic, presents to you for a health check-up
before she goes on a European tour that includes bush walking and travelling through
mountains. On examination, you notice a decline in her recent memory, as well as having
trouble in remembering new people's names and addresses. You perform a Mini mental state
exam ( MMSE) for her, the result of which is 24/30. You recommend that she cancel her tour
until further assessment is carried out because due to her current condition you think she is
unfit to travel alone. She refuses and tells you that she will stick to her plan. Which one of the
following is the next best step in management?

o Apply the case to Guardianship court.

B Admit her to the hospital.

C Contact the department of immigration and tell them about her condition.

D Send her to a nursing home.

E Do nothing.
X QA Apply the case to Guardianship court.

X QB Admit her to the hospital.

Contact the department of immigration and tell them about her condition.
X Oc

*o
Send her to a nursing home.

Do nothing.
O

Option E is correct

Patients with dementia thrive in familiar environments and by following a familiar routine. Adherence to routines
produces a sense of safety by knowing what is going to happen next and avoidance of confusion. Routines are
interrupted when traveling. For patients with cognitive impairment , traveling can be confusing. Being away from
familiar surroundings, eating and sleeping in unfamiliar places, disrupted sleep pattern, having to speak and
interact with strangers (such as airport and hotel staff) and having to follow directions that may not be fully
understood and become confusing for dementia patients. Confusion causes distress that may result in an
unusual , potentially catastrophic behavior. The first rule to consider about travelling is that no person with
dementia should ever travel unaccompanied because there are many decisions to make, directions to follow, and
unfamiliar surroundings to navigate that a person with dementia will find it overwhelming .

This woman has an MMSE score of 24, suggesting mild dementia . Examination also reveals cognitive function
impairment associated with this MMSE score.

She also needs thorough investigation for exclusion of organic and possibly reversible causes of dementia such
as hypothyroidism and vitamin B12 deficiency. Depression should be considered as a cause of pseudodementia
that can mimic dementia . Another important step is a full and formal assessment of her mental state and
psychological status because MMSE is a screening rather than an accurate diagnostic tool. MMSE should be
assessed and interpreted according to the patient’s educational status because while an MMSE score of 23 can
1L

be normal for a patient with 7ir grade education , it is clearly abnormal for another patient with college education.

In the meanwhile, this patient should be strongly advised to refrain from travelling alone, especially to unfamiliar
places . She should also be offered further assessment and follow-up; however, if she refuses to do so, nothing
more can be done . She is still a consenting adult and mild cognitive impairment will NOT preclude her from
making decisions for her.
i9:iid3ffl
^ ,
ti$ £houlcl ever travel unaccompanied because there are many decisions to make, directions to follow §9$, iy
unfamiliar surroundings to navigate that a person with dementia will find it overwhelming.

This woman has an MMSE score of 24, suggesting mild dementia. Examination also reveals cognitive function
impairment associated with this MMSE score.

She also needs thorough investigation for exclusion of organic and possibly reversible causes of dementia such
as hypothyroidism and vitamin B 12 deficiency. Depression should be considered as a cause of pseudodementia
that can mimic dementia . Another important step is a full and formal assessment of her mental state and
psychological status because MMSE is a screening rather than an accurate diagnostic tool. MMSE should be
assessed and interpreted according to the patient’s educational status because while an MMSE score of 23 can
th
be normal for a patient with 7 grade education, it is clearly abnormal for another patient with college education.

In the meanwhile, this patient should be strongly advised to refrain from travelling alone, especially to unfamiliar
places. She should also be offered further assessment and follow-up; however, if she refuses to do so, nothing
more can be done. She is still a consenting adult and mild cognitive impairment will NOT preclude her from
making decisions for her.

(Option A ) Applying for guardianship is not appropriate because this woman has only mild cognitive impairment
and is competent and can make decisions for herself.

(Option B ) Hospital admission is not necessary. Further assessment, if planned, is performed in outpatient
setting.

(Option C) Mandatory reporting to immigration department is not required. By traveling alone, this patient is not
putting anyone but herself at risk. Also, it is not a doctor’s duty or responsibility to provide such information to
such authorities. More importantly, immigration department has nothing to do with this woman’s travel
authorization.

(Option D ) An MMSE score of 24 or a recent decline in memory is not an indication for placing this patient in a
nursing home.

TOPIC REVIEW
Interpretation of MMSE score and the potential associated impairments is summarized in the following table:

Stage of cognitive Impairments in activities of


MMSE score
dysfunction daily life (ADL)
25-30 Probably normal • Probably none
• Problems coming up with the
right word or name
• Trouble remembering names
when introduced to new
people
• Having greater difficulty
performing tasks in social or
21 - 24 Mild dementia
work settings
• Forgetting material that one
has just read
• Losing or misplacing a
valuable object
• Increasing trouble with
planning or organizing
19:11 Fri 24 Apr • Forgetfulness of evee-ts@D 65% i
about one' s own personal
history
• Feeling moody or withdrawn,
especially in socially or
mentally challenging
situations
• Being unable to recall their
own address or telephone
number or the high school or
college from which they
graduated
• Confusion about where they
are or what day it is
• The need for help choosing
proper clothing for the
10- 20 Moderate dementia season or the occasion
• Trouble controlling bladder
and bowels in some
individuals
• Changes in sleep patterns,
such as sleeping during the
day and becoming restless at
night
• An increased risk of
wandering and becoming
lost
• Personality and behavioral
changes, including
suspiciousness and delusions
or compulsive, repetitive
behavior like hand- wringing
or tissue shredding

• Require full-time, around-


the- clock assistance with
daily personal care
• Lose awareness of recent
experiences as well as of
their surroundings
• Require high levels of
assistance with daily
activities and personal care
1 -9 Severe dementia
• Experience changes in
physical abilities, including
the ability to walk, sit and,
eventually, swallow
• Have increasing difficulty
communicating
• Become vulnerable to
infections, especially
oneumonia
Question ID 1188

Title Management of a 72-old woman with Alzheimer disease who is planning to travel alone

Notes
Q Flag as important

Font Size: A A A

Nathalie, 72 years old, is a patient of yours with Alzheimer disease. Today, she is in your office
for a health check before she goes on a European tour for bushwalking. On examination, she
has a mini- mental status exam ( MMSE) score of 21 / 30. Which one of the following is the most
appropriate action for you to take?

o Report her to immigration department.

B Tell her that she should cancel her trip as she is unfit.

C Telll her that she can go to the trip but should use her medications regularly.

Call her family and discuss the issue with them.

E Reassure that she can take the trip.


X A Report her to immigration department.

OB Tell her that she should cancel her trip as she is unfit.

X c Telll her that she can go to the trip but should use her medications regularly.

X QD Call her family and discuss the issue with them.

x OE Reassure that she can take the trip.

Option B is correct

As a rule, patients with dementia, regardless of the severity, should not travel alone. Even in its mildest form,
Alzheimer disease can be associated with forgetfulness and impaired visuospatial skills and appropriate
navigation and carries the risk of getting lost especially in an unfamiliar environment. Furthermore, bushwalking
includes treks in the nature which makes this tour even more risky for this patient.

Nathalie has an MMSE of 21 and can be categorized as having mild dementia. An MMSE score between 21 and
24 is associated with mild functional dependence. An MMSE score between 10 and 20 is seen in patients with
moderate Alzheimer disease and is associated with more immediate dependency, such as inability to drive,
difficulty with hygiene and shopping, and remote memory impairment. Severe disease (MMSE score under 10) is
associated with total dependence and the need for constant supervision. Motor impairments, notably gait and
balance impairment, incontinence, and myoclonus, develop at this late stage.

(Option A ) Mandatory reporting to immigration department is not required. It is not a doctor ’s duty or
responsibility to provide such information to authorities. On the other hand, immigration department has nothing
to do with this woman’s travel authorization.

(Option C ) There are approved medications such as donepezil, rivastigmine, or memantine, with modest effect
on dementia. They may delay the decline in cognition temporarily but cannot reverse the disease or prevent it
from progression. Even by taking her medications, Nathalie’s current cognitive status will not improve.

(Option D) Calling Nathalie’s family for discussing the issue with them is not a correct option because although
her cognition is mildly impaired, she is still competent adult and informing her family without her consent will be a
breach of confidentiality.

(Option E ) Reassurance is not appropriate because a trip long away from home while unaccompanied by a carer
is associated with risks to her health and well-being.
Question ID 1180

Title Ethical principle to consider in a patient refusing treatment

Notes
Q Flag as important

Font Size: A A A

A 38 -year - old man, who is a known case of schizophrenia and currently well-controlled on
ziprasidone, sustains bilateral fracture of his legs after he is hit by car as a pedestrian. He is
now in the emergency department and required surgery to fix the fractures, but he refuses
the surgery despite being fully informed of his condition and consequences of refusal. Which
one of the following concepts should guide further management?

Informed consent.
OA

B Utilitarianism.

C Patient's autonomy.

Patient's best interest.

E Incompetency.
A 38-year- old man, who is a known case of schizophrenia and currently well -controlled on
ziprasidone, sustains bilateral fracture of his legs after he is hit by car as a pedestrian. He is
now in the emergency department and required surgery to fix the fractures, but he refuses
the surgery despite being fully informed of his condition and consequences of refusal. Which
one of the following concepts should guide further management?

Informed consent.
X O

X QB Utilitarianism.

y Q Patient's autonomy.

Patient's best interest.


X O

X O Incompetency.

Option C is correct

Every competent adult has the right to refuse proposed treatment(s), no matter how the treatment is crucial or
beneficial for them to save their lives or health. This patient seems to understand the consequences of not
receiving treatment , yet he refuses it. In this case no treatment can be given based on the concept of autonomy.

Patient ’s autonomy is the most fundamental principle underlying all healthcare ethics . Autonomy grants every
competent adult patient the absolute right to do what he wishes with his own health care. The concept of
autonomy is fundamental to the entirety of the Australia legal system and has complete acceptance as an
operating principle of decision making.

Autonomy over one's own medical care is seen in the same light as freedom of religion, freedom from illegal
search and seizure, freedom of speech , and freedom of assembly. Patients have the right to refuse undesired
therapy, and they have the right to choose whether or not they will participate in experimentation. Each patient
has the right to have his wishes carried out even in the event that he loses consciousness or the capacity to
make decisions for himself. Autonomy represents a patient' s right to determine his or her own health-care
decisions .
Every competent adult has the right to refuse proposed treatment( s), no matter how the treatment is crucial or
beneficial for them to save their lives or health. This patient seems to understand the consequences of not
receiving treatment , yet he refuses it. In this case no treatment can be given based on the concept of autonomy.

Patient ’s autonomy is the most fundamental principle underlying all healthcare ethics . Autonomy grants every
competent adult patient the absolute right to do what he wishes with his own health care. The concept of
autonomy is fundamental to the entirety of the Australia legal system and has complete acceptance as an
operating principle of decision making.

Autonomy over one's own medical care is seen in the same light as freedom of religion, freedom from illegal
search and seizure, freedom of speech , and freedom of assembly. Patients have the right to refuse undesired
therapy, and they have the right to choose whether or not they will participate in experimentation. Each patient
has the right to have his wishes carried out even in the event that he loses consciousness or the capacity to
make decisions for himself. Autonomy represents a patient' s right to determine his or her own health-care
decisions.

(Option A ) Considering the concept of patient’s autonomy, informed consent is an essential part of medical
ethics. Consent means the patient’s approval before you proceed to touch, examine or treat them. Consent must
be informed , meaning the patient should be fully aware of treatment options, possible complications and material
risks.

(Option B ) Utilitarianism is a form of consequentialism stating that consequences of any action are the only
standard of right and wrong. A wrong action is good if the outcome is good and bad if the outcome is not good.
Action based on utilitarianism in this scenario means that the patient should be treated against his expressed will
to otherwise ( bad action), because the outcome of treatment is best for him (good outcome). This is not an
acceptable concept in medical ethics because it is against the wishes and wills of a competent patient and his/her
autonomy.

(Option D ) No form of a treatment can be pursued without the patient ’s agreement, even if the proposed therapy
is in the patient’s best interest. Although beneficence or doing what is good for people is a high aim and ethical
principle, autonomy is considered more important and takes precedence. Each patient has the right to refuse a
treatment even if that treatment has no adverse effects and will help them.

(Option E) Competency is a legal term. Only a court of law can decide if a patient is competent; however, for the
medical purposes as long as a patient appears to understand what is proposed and the consequences, criteria
for competency are met. Delirious state, dementia, intellectual disability, the effects of medications, psychotic
symptoms such as delusions and hallucinations and other forms of acute thought disorder makes a patient
incompetent for decision making. This patient has schizophrenia, but is well-controlled on medications and has
not thought disturbances at the moment, and cannot be considered incompetent and be overruled.
Question ID 1127

Title Management of a 24-year-old schizophrenic patient refusing treatment of his melanoma

Notes
Q Flag as important

Font Size: A A A

A 42-year- old patients of your, who suffers schizophrenia and is on controlled-release ziprasidone,
presents for a follow-up. During the visit you notice a mole on his face that is highly suspected to
be melanoma. You explain that the mole should be excised and send for pathology and further
follow-up will be required, but he refuses and says the mole will heal on itself. Which one of the
following is the most appropriate next step in management?

o Tell him about the risks of melanoma and benefits of treatment.

B Formal mental state examination.

C Refer him for a psychiatric opinion.

D Admit him involuntarily.

E Refer the case to Mental Health Tribunal.


A 42-year- old patients of your, who suffers schizophrenia and is on controlled-release ziprasidone,
presents for a follow-up. During the visit you notice a mole on his face that is highly suspected to
be melanoma. You explain that the mole should be excised and send for pathology and further
follow-up will be required, but he refuses and says the mole will heal on itself. Which one of the
following is the most appropriate next step in management?

o Tell him about the risks of melanoma and benefits of treatment.

x QB Formal mental state examination.

X c Refer him for a psychiatric opinion.

X QD Admit him involuntarily.

Refer the case to Mental Health Tribunal.


X O

Option A is correct

At common law and under some statutes, adults (people over 18) are presumed to be competent, although
it is possible to rebut the presumption by showing that an adult lacks competence. A functional test, settled
on by most common law jurisdictions, is used to assess the competency of a patient by healthcare
professionals to examine the ability of a patient to consent to or refuse a specific treatment. A patient is
competent if he/ she is able to:

• understand and retains treatment information


• believe the information
• weigh the information and reach a decision and communicate their decision
NOTE - even if the reasoning is not sound or even delusional, the doctor cannot consider the patient
incompetent to make decisions. Reasoning is a very important indicating the patient 's competency even if
the reasoning is irrational or delusional to the treating doctor. The following examples demonstrate how
the capacity to reason advocates the patients rights for autonomy:
[ Adult: Refusal of Treatment) [ 1994] 1 WLR 290 concerned a patient in the Broadmoor Asylum who refused
consent to having his gangrenous leg amputated. The doctors questioned his competence to make a
decision about treatment, given that he was suffering from a mental illness. The judge defined capacity as a
sufficient understanding of 'the nature, purpose and effects of the proffered [treatment]'. The mechanics of
understanding were split into three stages:
[ Adult: Refusal of Treatment) [ 1994] 1 WLR 290 concerned a patient in the Broadmoor Asylum who refused
consent to having his gangrenous leg amputated. The doctors questioned his competence to make a
decision about treatment, given that he was suffering from a mental illness. The judge defined capacity as a
sufficient understanding of 'the nature, purpose and effects of the proffered [treatment]’. The mechanics of
understanding were split into three stages:

• comprehending and retaining treatment information


• believing the information
• weighing it among other factors to reach a decision.
As the patient was able to complete these three steps he was found to have capacity, even though he was
suffering from schizophrenia and the delusional belief that he was a doctor.
(Adult: Refusal of Medical Treatment) [2002] EWHC 429, where a ventilator -dependent patient in an
intensive care unit sought to refuse treatment and be allowed to die. Some of the patient's doctors could
not accept her decision because they feared that she was experiencing a 'psychological regression' brought
on by her level of disability, her reaction to being totally dependent on others, her anger, and the effect of
being in an intensive care unit rather than a specialist unit.
The arguments against competence were rejected by the court. There was no evidence of psychological
regression, and the patient's lack of experience in rehabilitation did not go to the issue of whether she
understood the nature and effect of refusing treatment. It was said that doctors should not confuse the
question of capacity with their own emotional reaction to the patient's decision.
In contrast, incompetence is implied when the patient cannot reason for their refusal of treatment:

[ 1997] 2 FLR 426 concerned a woman who had consented to an emergency cesarean section but could not
consent to the insertion of a needle for the preliminary anaesthetic because of an acute needle phobia. The
court found that her phobia made her incompetent because it prevented her from being able to reason
through the issue of consenting to the anesthetic.
This patient has been offered treatment for a suspected melanoma and falsely believes that he does not
need treatment. In such situation, the most important step is discussing with him the risks melanoma can
pose to his health, treatment options and material risks associated with the treatment. He should then be
able to believe the information, weigh them and reach a decision and communicate his decision.
( Options B and C) This patient should not be assumed incompetence. However, if the patient, after
discussion, still refuses treatment against his best interest competence should be assessed
comprehensively. This may include:

• Assessment of the patient's understanding and beliefs through discussion with them
• Formal tests of cognitive capacity (such as the Standardized Mini-Mental Status Examination)
• Clinical psychological or neuropsychiatric assessment
• Consideration of corroborative history from other people; for example, family members and health
professionals such as the patient's general practitioner
( Option D ) Involuntary admission is the appropriate option if the patient is considered to be at immediate
risk of self harm or harm to others, or seem that due to a psychiatric condition is not capable of taking care
of himself/herself. None of these conditions are present and involuntary admission is not appropriate.

( Option E ) Referral to Mental Health Tribunal might be considered for patients when an order for
involuntary treatment is required, and if they are found to be incompetent for making decisions.
Question ID 1089

Title Consent for management of an unconscious patient with femoral fracture

Notes
Q Flag as important

Font Size: A A A

A 76-year- old lady is brought to the emergency department with fractured femoral neck by her
daughter who she lives with. She is unconscious and the “Do Not Resuscitate (DNR)” status is
unknown. Based on evaluations, she is in need for surgery for the fracture. Which one of the
following is the most appropriate action in this situation?

o Ask if there is a DNR order in place first.

B Proceed to the surgery without consent.

C Ask for consent from her daughter.

Ask for consent from guardianship tribunal.

F Refer her for palliative care.


X OB Proceed to the surgery without consent.

y Q) C Ask for consent from her daughter.

X D Ask for consent from guardianship tribunal.

Refer her for palliative care.


X O

Option C is correct

A "Do Not Resuscitate (DNR ) order means if the patient dies evident by cardiopulmonary arrest, the doctor does
not perform chest compressions, attempt electrical cardioversion, or use acute antiarrhythmic medications. In
other words, DNR is defined as not attempting any treatment once the patient is ‘dead ’, indicated by loss of
pulse, blood pressure and breathing. DNR orders cannot be considered valid and should not be followed unless
the patient losses all three components (pulse, blood pressure, breathing).

The most common misunderstanding about DNR is that being DNR must mean the patient is pre-terminal and is
just about to die.

Another common misunderstanding is that a person who is DNR should not get other aspects of an appropriate
medical treatment and care such as biopsies, surgeries, dialysis, etc.

DNR does not mean that the patient should let be dead just because a DNR order is in place. DNR just means
that the treating physician should consider death as the end point of life and attempts no further actions to
reverse it. So DNR is not valid until the patient is clinically dead evident by loss of pulse, blood pressure and the
ability to breathe spontaneously.

In this case scenario the patient is unconscious, not dead; therefore, asking if there is a DNR order in place
(option A ) will be inappropriate because even so, it does not apply in this situation. The patient, however, needs
surgery and consent is required for that matter. Since the patient is unconscious, her next of kin (daughter) can
give consent to the surgery on her behalf.

(Option B) Proceeding to the surgery without consent could be the correct answer in emergency cases, where
no one is present to consent on the patient's behalf, and to his/her best interest , and if failing to take prompt
action would put the patient’s health at serious risks. In such circumstances it is presumed that the carer would
consent to the patient ’s best interest if he or she was present.

(Option D) Referral of the case to guardianship tribunal is appropriate only if there is conflict between family
members or carers or when the family wish is in conflict with the patient’s best interest.

(Option E) Palliative care would be the correct answer if the patient or his/her caregiver /next of kin based on
sound grounds has refused active treatment.
Question ID 1088

Title Management of a patient with a DNR order and urgent need for surgery

Notes
Q Flag as important

Font Size: A A A

An 82-year - old man is admitted to the hospital with complicated bowel obstruction and
booked for emergecy surgery. He has a DNR order in place after he had a previous admission
for another reason last year. He still wishes to be DNR. What should you do about the
surgery?

No surgery can be done on a patient who is DNR.


OA

B Reverse the DNR order for the surgery.

C DNR is acceptable only if the surgery does not require intubation.

D Surgery is acceptable while DNR if an additional consent is signed.

E DNR does not preclude surgery; proceed with the surgery.


An 82-year - old man is admitted to the hospital with complicated bowel obstruction and
booked for emergecy surgery. He has a DNR order in place after he had a previous admission
for another reason last year. He still wishes to be DNR. What should you do about the
surgery?

X A No surgery can be done on a patient who is DNR.

X B Reverse the DNR order for the surgery.

Qc DNR is acceptable only if the surgery does not require intubation.


*
o Surgery is acceptable while DNR if an additional consent is signed.

DNR does not preclude surgery; proceed with the surgery.


X O

Option D is correct

A ' Do not resuscitate order ( DNR) is not meant to be a generalized limitation on all forms of therapy. You
1

can still intubate patients even if they are DNR.

DNR just means you are taking death as the endpoint of giving treatment. In the event of cardiopulmonary
arrest, do not give the additional therapy of cardiopulmonary resuscitation, defibrillation, and chest
compressions. DNR is not a general equivalent for withholding any other forms of therapy.
This patient needs surgery, and a DNR does not preclude this; therefore, proceeding with the surgery is the
most appropriate management option. However, like any other medical procedure, consent should be
obtained from the patient for the surgery if the patient is alert and competent to understand the treatment
options, their risks and complications and the consequences that might arise if treatment is withheld.
If the patient insists that DNR order be in place despite being informed of the consequences, no CPR should
be attempted in the event of cardiopulmonary arrest during the surgery or in the perioperative period.
Question ID 1087

Title Management of an 86-year -old woman with COPD, fractured femoral neck, and DNR order

Notes
Q Flag as important

Font Size: A A A

An 86-year-old woman with longstanding history of chronic obstructive pulmonary disease


(COPD) has an advance directive of DNR ( do not resuscitate ) order in place. She is now in the
emergency department with a femoral neck fracture sustained at home. She is drowsy and
disoriented. Her son and daughter are in the emergency department asking you to do whatever it
takes to treat her. Which one of the following should be your next appropriate action?

o Inform her son and daughter that she will be given all necessary treatment.

B Wait until she regains her consciousness and discuss the matter with her.

C Inform your senior doctor.

D Inform her son and daughter that there is nothing you can do as she is DNR.

E Refer the case to Guardianship Tribunal.


is rfie etajy department with a femoral neck fracture sustained at home. She is drowsy and - < > 55% St §
^disoriented
^ . Her son and daughter are in the emergency department asking you to do whatever it
takes to treat her. Which one of the following should be your next appropriate action?

y A Inform her son and daughter that she will be given all necessary treatment.

X B Wait until she regains her consciousness and discuss the matter with her.

X c Inform your senior doctor.

X D Inform her son and daughter that there is nothing you can do as she is DNR.

Refer the case to Guardianship Tribunal.


X O

Option A is correct

A "Do Not Resuscitate" (DNR) order means if the patient dies i.e. cardiopulmonary arrest, the treating
physician(s) does not perform chest compressions, attempt electrical cardioversion, or use acute antiarrhythmic
medications. In other words, DNR is defined as not attempting any treatment once the patient is dead, indicated
by loss of pulse, blood pressure and spontaneous breathing. DNR orders cannot be considered valid and should
not be followed unless the patient loses all of these.

The most common misunderstanding about DNR is that being DNR must mean the patient is preterminal and is
just about to die. Another common misunderstanding is that a person who is DNR should not get other aspects of
an appropriate medical treatment and care such as biopsies, surgeries, or dialysis.

DNR does not mean that the patient should let be dead just because a DNR order is in place. DNR just means
that the treating physician should consider death as the end point of life and take no further actions to reverse it.
Therefore, DNR is not valid until the patient is clinically dead evident by loss of pulse, blood pressure and the
ability to breath spontaneously.

This patient has a DNR order in place, but she is still clinically alive. She has a femoral neck fracture and the
DNR does not preclude an appropriate medical management of her current condition. She is not conscious now
but her next of kin can decide on her behalf. As they have given you the consent to proceed with treatment at
your discretion, the next appropriate action is telling them that you will give her all necessary treatments in her
best interest.

There is no need to inform the senior doctor, nor referring the case to Guardianship Tribunal, as the path is clear
and there is no conflict over the patient’s best interest in this scenario.
Question ID 1086

Title Management of an 82-year-old woman with critical illness and a DNR order in place

Notes
Q Flag as important

Font Size: A A A

Sophia is an 82- year - old patient of yours, who has been under your care for the past eight
years due to chronic obstructive pulmonary disease ( COPD). A few months back she informed
you that she had written a "Do Not Resuscitate ( DNR)" order. Today, he is brought to you by
her family member with complaint of severe dyspnea. She looks drowsy and confused. Her
family members appear quite concerned and ask you to treat her by any means. Which one of
the following is the most appropriate action?

OA Arrange a family meeting.

B Do not give any treatment as she has a DNR order in place.

C Admit her and assess for any need for ICU care.

D Refer the case to guardianship tribunal.

E Insert intravenous line and nasogastric tube and nothing more.


X (QA Arrange a family meeting.

X QB Do not give any treatment as she has a DNR order in place.

y c Admit her and assess for any need for ICU care.

X QD Refer the case to guardianship tribunal.

x Insert intravenous line and nasogastric tube and nothing more.


OE

Option C is correct

DNR means that patient should not receive cardiopulmonary resuscitation using chest compression, electric
cardioversion, or acute administration of antiarrhythmic medications when he or she sustains cardiopulmonary
arrest. This means absence of pulse, blood pressure and inability to breathe spontaneously which is not the case
in this scenario.

A common misunderstanding about DNR is that it is mistakenly interpreted as withholding appropriate treatment
from patients at end- stage conditions. Another common misunderstanding is that a person who is DNR should
not get other aspects of an appropriate medical treatment and care , such as biopsies, surgeries, or dialysis.

Since this patient is not clinically dead yet , DNR does not apply, and appropriate treatment, which is admitting her
and assessment for the need for any intensive care in the ICU, should be considered.

NOTE - When the patient has a cardiopulmonary arrest, it is always presumed that the patient has given
consent to CPR unless the patient has specifically and expressively refused this therapy in advance. In other
words, you do not need to ask for consent from anybody regarding CPR unless the patient is already DNR .
(Option A ) Since this patient is not subject to DNR yet , and the wishes of family are not against those of the
patient, a family meeting will be unnecessary.

(Option D ) Referral to Guardianship Tribunal would be the correct answer if there was any conflict between the
patient ’s best interests and the family members or between family members with equal levels of decision making
regarding the patient’s care.

(Option E) Just insertion of intravenous line and nasogastric tube, and not giving the patient appropriate medical
management is incorrect because neither the patient nor her family has requested withdrawal of treatment.
Question ID 1085

Title Management of a patient with a DNR order who is in need for intensive care

Notes
Q Flag as important

Font Size: A A A

A patient with Alzheimer disease from a skilled - nursing facility is admitted to the hospital for
severe upper gastrointestinal bleeding. The bleeding has not stopped despite multiple
transfusions, octreotide, proton ump inhibitors, and endoscopy. He needs monitoring and
evaluation in an intensive care unit. The patient has a DNR order in place by his legal
guardian. Which one of the following is correct regarding this situation?

o Reverse the DNR order and transfer to the ICU as needed.

B He cannot be DNR and be in the ICU.

C Transfer to the ICU as needed; it is okay to be DNR in the ICU.

D He can go to the ICU with the DNR but cannot go for surgery.

E He can go to the ICU with the DNR but cannot be intubated.


A patient with Alzheimer disease from a skilled - nursing facility is admitted to the hospital for
severe upper gastrointestinal bleeding. The bleeding has not stopped despite multiple
transfusions, octreotide, proton ump inhibitors, and endoscopy. He needs monitoring and
evaluation in an intensive care unit. The patient has a DNR order in place by his legal
guardian. Which one of the following is correct regarding this situation?

Reverse the DNR order and transfer to the ICU as needed.


*O
X QB He cannot be DNR and be in the ICU.

y Q Transfer to the ICU as needed; it is okay to be DNR in the ICU.

X QD He can go to the ICU with the DNR but cannot go for surgery.

x He can go to the ICU with the DNR but cannot be intubated.


OE

Option C is correct

A ‘Do Not Resuscitate’ or DNR order means no cardiopulmonary resuscitative efforts if the patients suffers a
cardiopulmonary arrest. There is no other automatic limitation in therapy besides this. DNR orders are often
confused with generalized withholding of care such as transfusion or ICU admission. DNR orders are not the
same as stopping the active management of the patient. DNR orders do not automatically mean that the patient
is immediately terminal or that there is an automatic assumption of palliative care only. This patient should be
transferred to the ICU, but in case CPR is indicated , it should be withheld.

Intubation, surgery, and any other form of treatment can be given to this patient as long as he is not in
cardiopulmonary arrest.

NOTE - When the patient has a cardiopulmonary arrest, it is always presumed that the patient has given
consent to CPR unless the patient has specifically and expressively refused this therapy in advance. In other
words, you do not need to ask for consent from anybody regarding CPR unless the patient is already DNR .
Question ID 1084

Title DNR order and reversible cardiopulmonary arrest during surgery

Notes
Q Flag as important

Font Size: A A A

Robert, aged 68 years, is in the operating theatre undergoing a colon resection due to colon
cancer. Prior to the surgery, the risks of the surgery was fully explained to him. After
discussion with the treating surgeon, he decided to sign a "Not for Resuscitation ( NFR)" form, if
anything happens to him during the surgery or afterwards. During the operation, he starts
bleeding suddenly and profusely and in a matter of seconds and before anything can be done,
he becomes pulseless and his heart stops beating. The blood pressure is not
recordable. Which one of the following is correct regarding this situation?

o Transfuse fluids and blood products, and resuscitate him as this has occurred as a
complication of the surgery.

B Do not resuscitate because the patient's blood pressure and pulse are not
recordable and he has arrested.

C You can give fluid, blood products and medications, but no chest compression or
electric cardioversion.

D Only administer intravenous fluids and blood products to restore circulating volume
but do not perform chest compression or cardioversion.

E DNR orders are not valid when they are not related to the underlying disease.
Robert, aged 68 years, is in the operating theatre undergoing a colon resection due to colon
cancer. Prior to the surgery, the risks of the surgery was fully explained to him. After
discussion with the treating surgeon, he decided to sign a "Not for Resuscitation ( NFR)" form, if
anything happens to him during the surgery or afterwards. During the operation, he starts
bleeding suddenly and profusely and in a matter of seconds and before anything can be done,
he becomes pulseless and his heart stops beating. The blood pressure is not
recordable. Which one of the following is correct regarding this situation?

X A Transfuse fluids and blood products, and resuscitate him as this has occurred as a
complication of the surgery.

not resuscitate because the patient's blood pressure and pulse are not
OB
recordable and he has arrested.

X c You can §ive fluid, blood products and medications, but no chest compression or
electric cardioversion.

X QD Only administer intravenous fluids and blood products to restore circulating volume
but do not perform chest compression or cardioversion.

DNR orders are not valid when they are not related to the underlying disease.
*O
Option B is correct

It is common to have patients presenting for surgery with a 'Do Not Resuscitate' (DNR ) order written in their files.
Physicians and patients suffer from misconception about the potential benefits and harms of resuscitation in the
operating room (OR ) , and even definition of resuscitation in the OR requires clarification prior to surgery.

Cardiopulmonary resuscitation (CPR ) in the OR has a very different prognosis than CPR in other areas. The
percentage of patients resuscitated in the OR, who return to their pre-CPR functioning, is 50-80% versus 4-14%
for those patients who are resuscitated in other areas. This difference is due to several factors such as the fact
that the arrest is always witnessed in the OR and the cause is often known that allows prompt effective
intervention targeted at the cause.

Another reason for this difference is that causes of arrest in the OR are often reversible effects of anesthesia or
hemorrhage, and not due primarily to the patient’s underlying disease. This fact makes physicians even more
uncomfortable with DNR orders in the OR because they may feel that their actions has led to the arrest, and they
are ethically obliged to resuscitate the patient, even if the patient has clearly expressed wishes to the contrary.
This is a significant misunderstanding. Physicians should be aware of the fact that competent patients or their
appropriate surrogates have the right to refuse medical procedures and care, even if the care is to counteract the
effect of previous medical intervention.

In this case scenario, the patient has been fully informed of the risks of the operation, yet he insists on a DNR
order to be in place. DNR means if the patient suffers a cardiopulmonary arrest, represented by undetectable
blood pressure and pulse and inability to breathe spontaneously, resuscitative measures such as chest
compression, electrical cardioversion, acute administration of antiarrhythmic drugs such as epinephrine, or
atropine must NOT be performed in respect to the patient ’s wish, even if the arrest could be reversed rapidly and
effectively. This holds true even if the arrest is just a simple consequence or complication of the procedure.

Just using intravenous fluids and blood products without establishing artificial circulation, at least initially, by
means such as chest compression is futile and not appropriate.

Again, once the patient arrests, measures considered as a part of basic life support (BLS) or advanced life
support (ALS) such as chest compression, assisted ventilation, use of medications, or electric cardioversion
cannot be used for the patient.
Question ID 1083

Title Approach to DNR order in a patient in need of life-saving dialysis

Notes
Q Flag as important

Font Size: A A A

You have a patient with severe multiple sclerosis that is advanced and progressive, who now
has developed renal failure secondary to diabetes. The patient is alert and has elected to put
the DNR order in place at her own discretion. Today, he has presented for follow-up and you
notive that he has a markedly elevated serum potassium of 8 mmol/ L. Which one of the
following is the most appropriate management of this patient?

o Dialysis cannot be done because of the DNR order.

B You can do the dialysis if the DNR is reversed for the procedure.

C Proceed with the dialysis; ignore the DNR order.

Give insulin and glucose until the DNR status is discussed with the family.

E Seek a court order to overrule the DNR order.


You have a patient with severe multiple sclerosis that is advanced and progressive, who now
has developed renal failure secondary to diabetes. The patient is alert and has elected to put
the DNR order in place at her own discretion. Today, he has presented for follow-up and you
notive that he has a markedly elevated serum potassium of 8 mmol/ L. Which one of the
following is the most appropriate management of this patient?

X 0> A Dialysis cannot be done because of the DNR order.

X QB You can do the dialysis if the DNR is reversed for the procedure.

Oc Proceed with the dialysis; ignore the DNR order.

x o Give insulin and glucose until the DNR status is discussed with the family.

Seek a court order to overrule the DNR order.


X O

Option C is correct

A "Do Not-Resuscitate' (DNR ) order is very specifically defined as refraining from resuscitative efforts, such as
chest compression , antiarrhythmic medications (e.g. amiodarone, atropine, adrenaline , etc.), and electrical
cardioversion in the event of the patient’s cardiopulmonary arrest. A DNR order has nothing to do with any other
forms of care the patient is receiving.

DNR order has no impact on the use of dialysis, and DNR order should be ignored when assessing the patient
for dialysis. Hyperkalemia is life-threatening. It is not reasonable to use an inferior therapy such as insulin and
glucose or resins for management of hyperkalemia when dialysis is indicated. In this scenario, DNR order should
be ignored and dialysis performed after obtaining the patient’s consent. The patient, however, still has every right
to refuse the dialysis . This is different from the DNR order.

This patient is awake, alert and able to understand his own medical condition ; therefore, the patient ’s family is not
relevant in the process of decision making if the patient has the capacity to understand his or her own medical
condition.
Question ID 1040

Title Response to the request of withholding information from a patient by a relative

Notes
Q Flag as important

Font Size: A A A

Nicole, 72 years, is one of your patients, who presented with a breast lump. Physical exam
finding of an inverted nipple and pea - au- de-orange made you highly suspicious of breast
cancer. You ordered a mammogram, the result of which confirmed the diagnosis. Her eldest
son calls and tells you that he thinks that her mother has breast cancer and asks you that you
do not tell her mother of diagnosis, if it turned out to be cancer, because she might become
depressed. Which one of the following would be the most appropriate action in this situation?

o Tell her son that you should meet Nicole alone and you have to inform her of the
diagnosis anyway.

B Tell her son to bring all family members for a family meeting.

C Arrange an appointment with the son for further discussion.

Call Nicole and ask her to bring a family member with her for the appointment.

E Follow the son's wish, as breaking the news might put Nicole at risk .
Nicole, 72 years, is one of your patients, who presented with a breast lump. Physical exam
finding of an inverted nipple and pea-au- de-orange made you highly suspicious of breast
cancer. You ordered a mammogram, the result of which confirmed the diagnosis. Her eldest
son calls and tells you that he thinks that her mother has breast cancer and asks you that you
do not tell her mother of diagnosis, if it turned out to be cancer, because she might become
depressed. Which one of the following would be the most appropriate action in this situation?

QA Tell her son that you should meet Nicole alone and you have to inform her of the
diagnosis anyway.

X B Tell her son to bring all family members for a family meeting.

X c Arrange an appointment with the son for further discussion.

Call Nicole and ask her to bring a family member with her for the appointment.
*O
Follow the son's wish, as breaking the news might put Nicole at risk.
X O

Option A is correct

As a rule, a patient is entitled to be informed of the diagnosis as soon as it is made. The information should not
be withheld on requests of relatives and carers. In this case, you should meet Nicole alone and you must inform
her of the diagnosis anyway.

It is Nicole’s decision whether to share any information about her diagnosis, health condition, or treatments
proposed.

(Options B , C and D) Any option suggesting family meetings or arranging a meeting with the requesting relative
is definitely wrong. In fact, telling about the patient' s medical information to a third party, irrespective of their
closeness or intimacy is an act of breach of doctor-patient confidentiality. Nicole does not need to bring a family
member unless she wishes so .

(Option E) There may be circumstances under which providing information could cause the patient harm, and
the treating doctor decides to withhold information from the patient. This is frequently referred to as therapeutic
privilege. Particular information may be withheld where the practitioner believes, on reasonable grounds, that
providing it may damage the patient’s health. The responsibility is on the practitioner to show that providing the
information would be reasonably likely to cause significant harm. This decision is made in very limited situations
by the treating doctor, not on a relative’s or carer ’s request.
Question ID 1039

Title Management of an independent 14-year- old girl requesting abortion

Notes
Q Flag as important

Font Size: A A A

Melisa, 14 years old, presents to your practice for termination of pregnancy at 13 weeks
gestation. She left home at the age of 12 years and has been living with her 20-year - old boy
friend for the last eight months against her parents' wishes. Her boyfriend left her after he
knew she is pregnant. Which one of the following is the most appropriate next step in her
management?

o Inform the Child Protection Service.

B Refuse her request because she is underage.

C Refer her for termination of pregnancy after consultation and obtaining informed
consent .

D Tell her that her parents should consent to pregnancy termination.

E Contact the police because the sexual relationship was illegal due to the age
difference.
Refuse her request because she is underage.
* QB
Qc Refer her for termination of pregnancy after consultation and obtaining informed
consent.

*o
Tell her that her parents should consent to pregnancy termination.

Contact the police because the sexual relationship was illegal due to the age
X 0E
difference.

Option C is correct

A child or minor is a person who is younger than the age of majority. In Australia , the age of majority is 18 years.
This is the age at which citizens can exercise all the civil rights available to Australians; however, a child of or
over 16 years of age may give consent for medical treatment. In certain situations, a child younger than 16 years
of age may give consent.

Based on Gillick competence principle, a minor (<16 years, but not younger than 13 years) is considered
competent to consent to treatment if:

• She/he lives independently of her/his parents (i.e. emancipated minor), AND


• She/he appears to have sufficient understanding and intelligence to enable her or him to understand fully
what is proposed and the risks and benefits.

If these criteria are met, a minor can consent to treatment without any requirements to informing a parent or
guardian or obtaining consent from them.

In summary, according to Gillick competence rule, a person under the age of 16 years is still able to give consent
for medical treatments including operative procedure on him/her providing that:

• He/she is 13 years of age or older, AND


• lives independent of his/her parents, AND
• he/she appears to fully understand the situation, treatment options, risks and benefits.
Although the relationship is illegal, because she is under the age of 16 years, they have been living together with
their own consent, and the boyfriend has not been a teacher, relative, or someone in whom the child puts her
trust because of their position. Providing this is the case , the police in Australia or the various child protection
authorities would not normally take action against either the girl or her parent. Informing the Child Protection
Services would be the correct answer and the next best step in management if the child was sexually assaulted.

Unless she gives permission to do so, it would be inappropriate for her parents to be advised of the pregnancy
and unnecessary for them to give consent to the procedure.
Question ID 1038

Title Management of a 17-year - old girl requesting an abortion after rape

Notes
Q Flag as important

Font Size: A A A

A 17-year - old girl presents to your clinic requesting an abortion at 12 weeks pregnancy. Her
pregnancy is the result of a rape that happened a while back in a party while she was drunk.
You are against abortion morally and think that abortion can be only ethical in cases of severe
congenital anomalies. Which one of the following should the most appropriate step in
management of this patient?

o Refer to another GP for a second opinion.

B Inform her parents.

C Inform the Sexual Assault Services.

D Refer to a tertiary hospital for further management.

E Perform the abortion despite your will.


A 17-year - old girl presents to your clinic requesting an abortion at 12 weeks pregnancy. Her
pregnancy is the result of a rape that happened a while back in a party while she was drunk.
You are against abortion morally and think that abortion can be only ethical in cases of severe
congenital anomalies. Which one of the following should the most appropriate step in
management of this patient?

S QA Refer to another GP for a second opinion.

X QB Inform her parents.

Qc Inform the Sexual Assault Services.


*
X QD Refer to a tertiary hospital for further management.

Perform the abortion despite your will.


X O
Option A is correct

The very first step in management of this patient is reporting the incidence of sexual assault to child protection
services. Any options suggesting mandatory reporting to child protection services would be the most appropriate
option.

According to current legislation in Australia, doctors, nurses and midwives are obliged to lodge a mandatory
report to the child protection services whenever they have formed a ‘reasonable belief” that a child under 18
years has been or is being sexually abused or assaulted WITHOUT taking into account whether the child is
dependent or independent ( mature ). Gillick competence rule does not exclude any child from being reported to
child protection services whatsoever if the case is sexual assault or abuse. All children younger than 18 years are
subject to mandatory reporting by the treating doctors, nurses and midwives, teachers, principals and the police
in such incidences.

The main point of the scenario appears to be the conflicting ethical issues regarding termination of the pregnancy
between the doctor and the patient. In instances where such conflicts exist and the doctor does not feel
comfortable in dealing with consultation regarding abortion, he/ she should advise the patient to see another GP
or a women’s health center as early as possible (ideally before 12 weeks gestation).

Termination of pregnancy should be performed by approved clinics and abortion service providers.

NOTE - Beyond a specific gestational age (20 weeks is South Australia , 22 weeks in Queensland, 18-20 weeks
in NSW, etc.) the termination of pregnancy might be subject to further review and assessment e.g., requiring
approval from a panel appointed by the Minister of Health in South Australia after 22 weeks.

(Option B ) This girl is older than 16 years and can consent to most medical treatments/procedures including
termination of pregnancy. No parental consent is required to do so, nor is there any obligation to inform them.

(Option C ) Sexual assault services are dedicated bodies that provide the victims of sexual assault with
counselling services, forensic examinations and psychological care. While offering referral is appropriate,
informing them without the patient’s consent is a breach of doctor-patient confidentiality.

(Option D ) Referral for further management is considered after discussion with the patient about the procedure
and obtaining informed consent for referral after the patient is fully informed of the risks and potential
complications. While the doctor is against termination of pregnancy, such discussion should be taken over by a
third party ( e.g., another doctor) who is impartial and does not feel uncomfortable with the issue of termination of
pregnancy.

(Option E) The doctor is not to obliged to become involved in abortion if it is against his/her ethical principles.
Question 1037
ID

Title Management of a dependent 13 -year - old girl requesting termination of pregnancy after
rape

Notes
Q Flag as important

Font Size: A A A

A 13-year- old girl presents to your clinic asking for termination of pregnancy. Three weeks ago, she
was in a party where she was coerced to have unwanted sex with a stranger. Yesterday, using a
home pregnancy test , she realized she is pregnant. She is a school girl and lives with her parents.
Which one of the following is the most appropriate next step in management?

Inform the Child Protection Service.


OA

B Inform her parents of the event.

C Terminate the pregnancy.

Inform the police.

E Inform the Sexual Assault Services.


A 13-year- old girl presents to your clinic asking for termination of pregnancy. Three weeks ago, she
was in a party where she was coerced to have unwanted sex with a stranger. Yesterday, using a
home pregnancy test , she realized she is pregnant. She is a school girl and lives with her parents.
Which one of the following is the most appropriate next step in management?

QA Inform the Child Protection Service.

x QB Inform her parents of the event.

X c Terminate the pregnancy.

*o
Inform the police.

x o Inform the Sexual Assault Services.


According to current legislation in Australia, doctors, nurses and midwives are obliged to lodge a mandatory
report to the child protection services whenever they have formed a ‘reasonable belief” that a child under 18
years has been or is being sexually abused or assaulted.

NOTE - Mandatory reporting in cases of sexual assault DOES NOT take into account whether the child is
dependent or independent (mature). Gillick competence rule does not exclude any child from being reported
to child protection services whatsoever. All children younger than 18 years are subject to mandatory reporting
by the treating doctors, nurses and midwives, teachers, principals and the police.
Gillick competence rule, on the other hand, applies for termination of pregnancy, and a Gilick-competent minor
can request an abortion without parental consent.

Based on Gillick competence principle, a minor (<16 years, but not younger than 13 years) is considered
competent to consent to treatment if:

• she/he lives independently of her/his parents (i.e. emancipated minor) , AND


• she/he appears to have sufficient understanding and intelligence to enable her or him to understand fully
what is proposed and the risks and benefits.

If these criteria are met, a minor can consent to treatment without any requirements to inform a parent or
guardian or obtaining consent from them .

To put it in a more clear and straightforward way, always approach a minor as a consenting adult if:

• he/she is 13 years of age or older, AND


• he/she is independent of his/her parents, AND
• he/she appears to fully understand the situation, treatment options, risks and benefits.
(Options B and C ) This child does not fulfill criteria for termination of pregnancy without parental consent or
informing . Although she is 13 ( minimum age required), she lives with her parents and cannot consent to
termination of pregnancy. Regulations related to termination of pregnancy in a minor varies in different states but
generally parents should consent or at least be informed (according to the state regulations); however, a child
aged 13 or more can still refuse her/his parents being informed. As the healthcare professional involved in the
treatment , you should respect her /his wish in this regard. No mandatory reporting to the parents is required. The
decision as to whether the parents are informed and how to be so is the responsibility of the child protection
service and not of the primary healthcare provider.

If she is willing to terminate the pregnancy and does not want her parents/carer/guardian involved, a court order,
often issued by Children’s Court is required before proceeding to termination of pregnancy.

(Option D ) Informing the police is only acceptable if the victim consents to, or when there is immediate threat to
the victim’s health. Informing the police is not helpful if the child is not willing to disclose the assault to the police
because police cannot take action if there is no evidence; however, in most cases there is close collaboration
between the police and the child protection service.

(Option E) Sexual assault services are dedicated bodies that provide the victims of sexual assault with
counselling services, forensic examinations and psychological care. Although it is advisable that sexual assault
victims be counselled about the benefits of such services and offered referral, no mandatory reporting to sexual
assault services is required.
Question ID 1036

Title Management of a independent 13-year -old girl after she was raped

Notes
Q Flag as important

Font Size: A A A

A 13-year - old girl comes to your GP clinic asking for options regarding abortion. She left home
12 months ago and is living with his 18-year - old boyfriend. Last evening, she went to a party
where she became drunk and was forced to have unwanted sex with a man. She wants to
know if she could have an abortion in case she gets pregnant. On examination, there is no
evidence of trauma. Which one of the following should be notified first?

o Sexual assault service.

B The police.

C Her parents.

D School.

E Child protection services.


The key point to appreciate and fully absorb is the legal concept of ‘child’ in the term ‘mature child’. No matter
how mature or independent they might be, they are children as long as they are younger than 18 years and
reporting any alleged abuse/assault is a ‘must ’.

-
NOTE This is different from incidences when a mature child voluntarily involves in a sensual sexual
relationship. In such cases no mandatory reporting is required if the child is 13 years or older.
When a belief of child sexual abuse or assault is formed, the practitioner should discuss the mandatory reporting
requirements with the mature minor and include the following in the discussion:

• How the minor would like a report to be progressed


• Their preferences for alerting (or not) their carers/parents
• Their preference for informing (or not) the Police
• If the minor views themselves at any ongoing risk of (further) sexual abuse
• Any alerts for the child protection service or the police
• Any further information the mature minor would like to have included in the report
Child protection services should make a mandatory report to the police; however, if the mature minor in
discussion with the mandatory reporter, determines they do not want the police to be involved, this information
needs to be included in the report. Child protection services and the police will take this into account whenever
there is no ongoing risk to this child or any other child.

-
NOTE It is always appropriate and advisable to persuade or encourage a mature minor to inform her
parents/cares or to allow the doctor to do so on their behalf, but if the mature refuse to do so, it should be
avoided as it unnecessarily breaches the doctor- patient confidentiality.
Her school (option D) is not required to be informed in any case; even if the child is dependent and not mature .

Sexual assault services ( option A ) are dedicated bodies that provide the victims of sexual assault with
counselling services, forensic examinations and psychological care. Although it is advisable that sexual assault
victims be counselled about the benefits of such services and offered referral, no mandatory reporting to sexual
assault services is required.

TOPIC REVIEW
A child or minor is a person who is younger than the age of majority. In Australia the age of majority is 18 years.
This is the age at which citizens can exercise all the civil rights available to Australians; however, a child of or
over 16 years of age can give consent for medical treatment. In certain situations, a child younger than 16 years
of age may give consent as well if he/she is considered competent by Gillick competence rule.

Based on Gillick competence principle, a minor (<16 years, but not younger than 13 years) is considered
competent to consent to treatment if:

• she/he lives independently of her/his parents (i.e. emancipated minor), AND


• she/he appears to have sufficient understanding and intelligence to enable her or him to understand fully
what is proposed and the risks and benefits.

If these criteria are met, a minor can consent to treatment without any requirements to informing a parent or
guardian or obtaining consent from them .

To put it in a more clear and straightforward way, always approach a minor as a consenting adult if:

1. she/he is 13 years of age or older, AND


2. she/he lives independent of his/her parents, AND
3. she/he appears to fully understand the situation, treatment options and risks and benefits.
n .^ sexual assault service . T VST OD /o 7 J'

x OB The police.

X Qc Her parents.

x QD School.

QE Child protection services.

Option E is correct

According to current legislation in Australia, doctors, nurses and midwives are obliged to lodge a mandatory
report to the child protection services whenever they have formed a ‘reasonable belief ” that a child under 18
years has been or is being sexually abused or assaulted. This holds valid for all peoples younger than 18 years
of age regardless of whether they are dependent or independent (mature) minors; therefore , the next step in
management in this scenario is reporting to child protection services.

The doctor, to whom a mature minor discloses the sexual assault, should initially inform the child of legal
requirements of mandatory reporting and the limitations on doctor-patient confidentiality.

The key point to appreciate and fully absorb is the legal concept of ‘child’ in the term ‘mature child’. No matter
how mature or independent they might be, they are children as long as they are younger than 18 years and
reporting any alleged abuse/assault is a ‘must ’.

-
NOTE This is different from incidences when a mature child voluntarily involves in a sensual sexual
relationship. In such cases no mandatory reporting is required if the child is 13 years or older.
When a belief of child sexual abuse or assault is formed, the practitioner should discuss the mandatory reporting
requirements with the mature minor and include the following in the discussion:

• How the minor would like a report to be progressed


• Their preferences for alerting (or not) their carers/parents
• Their preference for informing (or not) the Police
• If the minor views themselves at any ongoing risk of (further) sexual abuse
• Any alerts for the child protection service or the police
• Any further information the mature minor would like to have included in the report
Child protection services should make a mandatory report to the police; however, if the mature minor in
discussion with the mandatory reporter, determines they do not want the police to be involved, this information
needs to be included in the report. Child protection services and the police will take this into account whenever
there is no ongoing risk to this child or any other child.
Question ID 1035

Title Management of a child with suspected child abuse

Notes
Q Flag as important

Font Size: A A A

A 6-year- old child is presented with multiple bruises. Based on reasonable grounds, you form
the idea that physical abuse has occured to this child. Which one of the following is the most
appropriate next action you should take?

o Full blood exam.

B PT, APTT.

C X- ray.

0 Take photographs of the lesions.

E Notify the Child Protection Service.


Full blood exam.
X O

x QB PT, APTT.

x Oc x ray -
'

Take photographs of the lesions.


^

X o Notify the Child Protection Service.

Option D is correct

When a healthcare professional, based on reasonable grounds, forms a belief that child abuse has occurred
immediate action should be taken.

The priorities in dealing with child abuse are:


1. To diagnose , treat and document the child injuries
2. To notify and involve the Child Protection Services immediately
3. To provide, when consent is given, a verbal or written report to Child Protection Service and the Police (this is
different from notifying the Child Protection Services on perceived child abuse)
This child does not appear to be in immediate need for treatment as the first priority; therefore, taking
photographs of the lesions to document them is the next best step in management.

-
NOTE Notifying the Child Protection Services does not need any consent from the parents, care or
guardian. In fact, it is advisable that the doctor withhold from the parent that he/ she has made a notification if
it is believed that the accompanying parent has been involved in child abuse. However, the doctor must
establish that consent has been given (by one of the parents or the child's legal guardian) to perform a clinical
examination and to provide a report (not notification) to Child Protection and the Police . Ideally this should be
in writing. If consent is unobtainable, the child should only be examined if a medical emergency exists .
(Option A , B and C ) Full blood exam and coagulation profile would be needed if the bruises were considered to
have been caused by a medical condition with hemorrhage tendency. These tests might later be indicated for
further assessment after the bruised are proved not to be a consequence of injuries. X-ray exam might be
considered somewhere during assessment process if indicated. It is not a priority now.

(Option E) Notification to child protection services comes next after documenting the injuries and treating them.
Question ID 990

Title Approach to a parent refusing vaccination of her 6- month- old daughter

Notes
Q Flag as important

Font Size: A A A

A mother brings her 6- month- old daughter for advice regarding vaccination of her baby. The
child has not received any vaccine so far because the mother had believed it is more natural,
but she was a little concerned after she read in an article that vaccination is beneficial
for children. You explain to her the benefits of the child being vaccinated. At the end, she
decides not to vaccinate her child. Which one of the following is the most appropriate
approach in this situation?

o Inform child protection services.

B Call the police as she is putting the baby's health at risk.

C Inform the local community council.

D Respect her decision.

E Apply for guardianship to the guardianship court.


A mother brings her 6- month- old daughter for advice regarding vaccination of her baby. The
child has not received any vaccine so far because the mother had believed it is more natural,
but she was a little concerned after she read in an article that vaccination is beneficial
for children. You explain to her the benefits of the child being vaccinated. At the end, she
decides not to vaccinate her child. Which one of the following is the most appropriate
approach in this situation?

X A Inform child protection services.

X B Call the police as she is putting the baby's health at risk.

X c Inform the local community council.

o Respect her decision.

xO Apply for guardianship to the guardianship court.

Option D is correct

According to current regulations in Australia , vaccination is not compulsory and parents can choose not to
vaccinate their children. Physicians are required to fully explain the benefits and risks of vaccination to parents
and respect their wishes if they refuse vaccination of their children.

Any option suggesting reporting of such parents to authorities such as child protection services, police, court, etc
is incorrect.
Question ID 976

Title Exceptions to doctor - patient confidentiality

Notes
Q Flag as important

Font Size: A A A

All of the following situations allow the doctor -patient confidentiality to be breached except:

o When the patient consents to allow personal details to be revealed to a third party.

B If there are other health professionals who have a legitimate therapeutic interest in
the care of the patient including medical students.

C If there are other health professionals who have a legitimate therapeutic interest in
the care of the patient excluding medical students.

D If there is overriding public interest.

E Where disclosure of the information is required or permitted by operation of the


law.
19: 2 Fri 2 Apr when the patient consents to allow personal details to be revealed to a thirchp ®r .%
£
^ ^ *
If there are other health professionals who have a legitimate therapeutic interest in
^ B
the care of the patient including medical students.

X c If there are other health professionals who have a legitimate therapeutic interest in
the care of the patient excluding medical students.

*o
If there is overriding public interest.

x o Where disclosure of the information is required or permitted by operation of the


law.

Option B is correct

For a proper doctor-patient relationship it is important to ensure that information provided by patients to the
treating doctor will remain strictly confidential.

The general rule is that doctors may not, without the consent of their patients, disclose to any third party
information acquired in the course of their professional relationship. This rule of confidentiality extends also to
disclosure to family members.

However, there are exceptions where confidentiality can be breached. These exceptions are as follows:
Where the patient gives valid consent for his/her medical information to be revealed to a third party - e.g. the
patient asks you to reveal his medical information to his/her employer, insurance company, etc.

Sharing information in the healthcare team - in many health care situations, consent for sharing confidential
information between members of the ‘health-care team’ is implied and it is presumed that patients know and
accept that this will happen. These members include other health professionals who have a legitimate therapeutic
interest in the care of the patient. Medical students cannot be considered legitimate in this sense and are
excluded , unless the patient consents to . In fact, medical students should be considered thirds parties.

Exceptions established by law - these include the notification of infectious diseases, births and deaths, and
deaths reportable to the coroner. In some states, doctors are obliged to notify the relevant registration authority if
a health-care professional, who is a patient, is ill and the community is believed to be at risk ; this exception is
backed by immunity from civil action.

Overriding community interest - when community interest overrides that of the patient, disclosure of the patient’s
medical information is not considered a breach of confidentiality. An example is when a doctor advise the police
of a patient they believe should not be driving a motor vehicle, or a psychiatrist believes a patient is a serious
threat to others.
Question ID 975

Title Approach to a non-English speaker patient

Notes
Q Flag as important

Font Size: A A A

A 34-year - old construction worker presents to your clinic after a foreign body entered his left
eye while working. He is from Algeria and does not speak English. His supervisor is
accompanying him in the visiting room and asks if he could translate for the patient. Which
one of the following is the most appropriate action to take in this situation?

o Ask for an accredited translator to be present.

B Bring an accredited interpreter on the phone.

C Ask the supervisor to leave the room and do not intervene because the patient's
privacy could be breached.

Ask the patient if he wants his supervisor to do the interpretation.

E Let the supervisor to do the interpretation.


X A Ask for an accredited translator to be present.

y QB Bring an accredited interpreter on the phone.

X Qc Ask the supervisor to leave the room and do not intervene because the patient's
privacy could be breached.

X O Ask the patient if he wants his supervisor to do the interpretation.

Let the supervisor to do the interpretation.


X O

Option B is correct

Language differences pose a challenging situation on doctor-patient relationship where the treating doctor and
the patient speak different languages. The problem is prominent in countries such as Australia where the number
of those unable to efficiently speak and/or understand English is considerable owing to high rate of immigration
from overseas.

It is essential that in situations where a patient has some difficulty or uncertainty in understanding English, a
qualified healthcare interpreter is used. The fact that a healthcare interpreter / translator has been used should be
noted in the patient ’s medical records. Most consent forms have a space for statement that the translator has
translated the contents of the form and the information given by the patient.

It should be noted that a member of the patient’s family, a friend or another non-accredited person should not, in
general, act as interpreter, as both legal and ethical questions could be raised about the validity of any consent
obtained. An exception is when the medical issue is minor and use of a close friend or family member is the
expressed wish of the patient.

In this case scenario, the treating doctor should ask for an accredited healthcare interpreter to be present;
however, since it is often impossible to have a qualified interpreter available on the spot, the Telephone
Interpreting Service (TIS) should be used as the most convenient means of accessing to an interpreter. This
service has a dedicated telephone number for doctors in private practices and the service is free when doctors
are providing care. The service is claimable under Medicare to Australian citizens or permanent residents.

If an unqualified interpreter has been used, for example in an emergency, a qualified interpreter should be
obtained as soon as possible to ensure that the patient has understood what has taken place.

It is not appropriate to use the supervisor to translate despite the fact that the patient might have consented to it.
When the translator is present, the doctor could reliably ask the patient about his preference is he prefers the
supervisor to be present.
Question ID 958

Title Dealing with a case of self-harm by a 16-year -old girl

Notes
Q Flag as important

Font Size: A A A

You are assessing a 16-year - old girl, who has cut her wrist intentionally. You talk to her and
after a thorough assessment you are reassured that she is not suicidal, nor does she have any
suicidal ideations at the moment. Her parents separated two years ago. Currently, she is living
with her mother, but she wishes to live with her father. Both her parents and the school
psychologist are concerned about her and insist to be informed of her condition. Which one
of them should you inform?

o Only the father.

B Only the mother.

C Both parents.

Only the school psychologist.

E None of them.
X A Only the father.

X B Only the mother.

X c Both parents.

X O Only the school psychologist.

o None of them.

Option E is correct

The case represents self-mutilation in the absence of an intention to die or suicidal thoughts. This is termed non-
suicidal self-injurious behavior. The behavior is purely for non-suicidal reasons, either to relieve distress or to
make a change in others or the environment, or for anxiety relief.

This girl is 16 years old and considered an adult in most areas of healthcare. She has a legal right to confidential
healthcare.

The exception is when there is a significant concern of risk to self or others. This patient , based on the opinion of
a healthcare professional, is not suicidal and does not pose any harm to herself, at least for now; therefore, she is
excluded from this exception. Her current problem should not be informed to anyone without her expressed
consent ; however, she should be encouraged to seek help and counselling from professionals (e.g. , school
psychologist), or her supporting resources such as her parent.

Confidentiality is a means of providing the client with safety and privacy and therefore, protects client autonomy.
For this reason, any limitation on the degree of confidentiality is likely to diminish the effectiveness of counselling.

It should be noted though that if the patient was acutely suicidal, she lacked competence and involuntary actions
could have been considered.

-
NOTE Notifying the parents of a dependent minor cannot be performed without their consent. Should any
dispute arise between the duty of the health professional and the child ’s refusal of parental notification , the
issue should be referred to an authorized third party such as a court.
Question ID 934

Title Identifying the most likely cause of genital wart in a 2-year- old girl

Notes
Q Flag as important

Font Size: A A A

A mother brings her two- year- old daughter to your clinic because she is concerned about a lesion
on her child' s external genitalia. On examination , the lesions turn out to be genital wart (condyloma
acuminata ). The mother has the history of treatment for a CIN1 cervical dysplasia. Which one of
the following can be the most likely cause of her genital wart?

o Perinatal infection from the mother.

B Sexual abuse.

C Primary infection.

D Breastfeeding.

E Autoinoculation.
o Perinatal infection from the mother.

X QB Sexual abuse.

X c Primary infection.

X D Breastfeeding.

Autoinoculation.
X O

Option A is correct

Sexually transmissible diseases (STIs) are rarely seen in cases of sexual child abuse but if present, strongly
suggest the possibility. In other words, a child with STI has suffered sexual abuse until proven otherwise ; this
however, does not mean that sexual abuse is the most likely explanation.

Anogenital warts or condyloma acuminata are caused by the human papilloma virus (HPV).

Studies indicate that in adults, genital HPV infections are primarily sexually transmitted. In children, the mode of
transmission of HPV infection is not as straightforward. Sexual transmission is recognized as a possibility in
children , but other possible modes of transmission have been documented as well.

In summary, the modes of HPV transmission in children include:

Sexual abuse

• Oral-genital contact
• Genital-genital contact
• Genital-anal contact
• Fondling
• Digital penetration of the vagina or anus
Nonsexual transmission

• Autoinoculation
• Direct contact with caretaker
• Contact with objects or surfaces contaminated with HPV
19:23, Fri 2,4 Apr
Sexual abuse -r < > 56%
§ + •

• Oral-genital contact
• Genital-genital contact
• Genital-anal contact
• Fondling
• Digital penetration of the vagina or anus
Nonsexual transmission

• Autoinoculation
• Direct contact with caretaker
• Contact with objects or surfaces contaminated with HPV
Vertical transmission (from mother to infant )

• Via bloodstream prior to birth


• During vaginal delivery through infected birth canal
• Via cesarean section with or without early rupture of membranes
Sexual abuse must never be eliminated when considering possible modes of transmission for anogenital HPV.
Many forms of sexual abuse can result in transmission of HPV, including genital-genital contact, genital-anal
contact , oral-genital contact, fondling, and digital anal/genital penetration.

However, Adams' (2001) classification scale for evaluating medical findings of suspected sexual abuse lists
anogenital warts/condyloma in a child younger than two years of age as a nonspecific finding for sexual abuse. In
such cases perinatal transmission must be considered first as the most likely explanation.

Vertical transmission of the HPV virus does not mean that warts must be present at birth or shortly after birth.
HPV is a latent virus and can reside in the skin and mucous membranes without causing warts . The warts may
not appear until months or even years after birth. Some authors believe the time between infection and the
presentation can be as long as five years. Some believe in a shorter period of up to two years. In general, vertical
transmission of HPV can still be the main cause even if lesions first appear years after birth.

Vertical transmission can occur through the bloodstream prior to birth, or at the time of birth as the infant passes
through the infected birth canal. Delivery via cesarean section ( with or without premature rupture of membrane)
does not eliminate the possibility of vertical transmission of HPV. There are even reports of congenital condyloma
after caesarean section without premature rupture of membranes.

Some authors also believe that that HPV transmission can occur in utero through semen, ascending infection
from the mother's genital tract, or transplacentally.

Anogenital warts (HPV) also can be transmitted via autoinoculation. Children with a common wart on their hands
or elsewhere on their body can transmit the virus by touching their warts and then touching their own genitals.

Non-sexual transmission can also occur from direct contact with caretaker contaminated with genital HPV or
common warts. For example, caretakers with genital warts who touch or scratch their genitals and then, without
washing their hands, change a baby's diaper or assist a child with toileting/bathing may transmit the virus to the
child's genitals. HPV transmission via contact with contaminated objects or surfaces is also possible.

The mother has been treated with CIN1. Although HPV serotypes associated with cervical cancer are different
from thosecausing anogenital warts, presence of cervical neoplasia could suggest co-infections with other types
of HPV as well. In this child, with the mother ’s possible infection and the child’s age, the most likely cause to the
child’s anogenital warts appears to be perinatal infection from the mother.

If the child was older the likelihood of sexual abuse would be more pronounced, as perinatal infection must have
presented by 2 years of age, as most authors believe. Although not a rule, the younger the child, the more likely
the HPV infections is due to perinatal infections rather than sexual abuse.
Question ID 929

Title Approach to a 22-year - old patient refusing dialysis

Notes
Q Flag as important

Font Size: A A A

A 22 -year - old Aboriginal man is in the waiting list for kideny transplant due to end -stage renal
disease ( ESRD). In the meanwhile, he is on dialysis three times a week. He has presented to
you as his treating physician and says he does not want to undergo dialysis and wants to
withdraw from treatment. Which one of the following would be the most appropriate action
to take?

o Arrange for a donor for him.

B Ignore his wish and treat him.

c Arrange a family meeting.

D Discuss his decision with him to make sure he understands the consequences.

F Refer the case to the court.


A 22 -year - old Aboriginal man is in the waiting list for kideny transplant due to end - stage renal
disease ( ESRD). In the meanwhile, he is on dialysis three times a week. He has presented to
you as his treating physician and says he does not want to undergo dialysis and wants to
withdraw from treatment. Which one of the following would be the most appropriate action
to take?

X A Arrange for a donor for him.

X QB Ignore his wish and treat him.

X c Arrange a family meeting.

Discuss his decision with him to make sure he understands the consequences.
O

Refer the case to the court.


X O

Option D is correct

Patient autonomy is the cornerstone of all healthcare ethics. Every competent adult has the absolute right to do
what they desire with their own health and life. Competency is a legal term and not determined by the healthcare
workers. Capacity, which is a different term, is what used instead in medical decision-making process. These two,
however, can be used interchangeably.

When a patient decides not to accept or withdraw from a treatment, the most appropriate next step is always a
full discussion with the patient about the potential consequences of his/her decision and making sure that they
understand them .

(Option A ) Arranging for a donor just because he does not want to go on with the treatment is not appropriate.
Every patient should follow specific protocols.

(Option B ) Ignoring a competent patient 's expressed wish and acting differently is an act of battery and
punishable by the law.

(Options C and E ) Arranging a family meeting for discussion about a competent patient’s wish is not appropriate.
Neither is referring the case to the court because the law is quite straightforward on this matter.
Question ID 916

Title Management of suspected elderly abuse in a 82-year -old man

Notes
Q Flag as important

Font Size: A A A

During visiting an 82-year - old man for an upper respiratory tract infection, you notice multiple
bruises on different body parts. He lives with his daughter and her boyfriend, and confides in
you that the bruises are the result of being physically abused by her daughter's boyfriend. He
adamantly insists that you should not inform the police or any other authorities because he
can deal with his problem by himself. You perform a mental status exam, including a 'serial
seven' the result of which is normal. Which one of the following would be the most
appropriate management option in this situation?

o Discharge him home after management of the bruises.

B Obey his wish but arrange for regular follow -ups.

C Ask him to see a social worker before he leaves the hospital.

D Inform the daughter about abuse so that she knows what is happening with her
father.

E Notify the police immediately.


During visiting an 82-year -old man for an upper respiratory tract infection, you notice multiple
bruises on different body parts. He lives with his daughter and her boyfriend, and confides in
you that the bruises are the result of being physically abused by her daughter's boyfriend. He
adamantly insists that you should not inform the police or any other authorities because he
can deal with his problem by himself. You perform a mental status exam, including a 'serial
seven' the result of which is normal. Which one of the following would be the most
appropriate management option in this situation?

X QA Discharge him home after management of the bruises.

Obey his wish but arrange for regular follow -ups.


^ OB

X 0c
( Ask him to see a social worker before he leaves the hospital.

X D Inform the daughter about abuse so that she knows what is happening with her
father.

Notify the police immediately.


*O

Option B is correct

Elder abuse must be considered by any health practitioner seeing elderly patient as they have an essential role in
the recognition, assessment, understanding and management of elder abuse and neglect. Once faced with
elderly abuse, the first thing to consider is to assess if the patient has the capacity to make decisions ( as in this
case where the mental status of the patient has been evaluated ).

The elderly should be consulted about the criminal nature of abuse and that it is unacceptable and there is
always means to prevent it. They should be made aware of they legal rights and that they can seek legal action
and protection if they wish so at any time.

If the elderly patient has the decision-making capacity and refuses any intervention, their decisions must be
respected, but he/ she should be advised to contact you or other support agencies for help in the future . The
status of the elderly should be checked through regular follow-ups.
Question ID 913

Title Management of a suspected case of child abuse

Notes
Q Flag as important

Font Size: A A A

You are a resident at the Emergency Department. An angry father approaches you because
the social worker has been asking him if he has punished his child physically. The child is
five years old and has been in the Emergency Department four times this year with several
episodes of trauma not consistent with the alleged history given by the parents. Today, the
child is brought with a complaint of 'slipping into a hot bathtub' with a burn wound on his legs.
The father threatens to sue you and says 'how dare you think that about me, I love my son!'
Which one of the following would be the most appropriate next step in management?

o Admit the child to remove him from the possibly dangerous environment.

B Call the police.

C Ask the parents if there has been any abuse.

Speak to the wife privately about possible child abuse.

E Report the family to child protective services.


You are a resident at the Emergency Department. An angry father approaches you because
the social worker has been asking him if he has punished his child physically. The child is
five years old and has been in the Emergency Department four times this year with several
episodes of trauma not consistent with the alleged history given by the parents. Today, the
child is brought with a complaint of 'slipping into a hot bathtub' with a burn wound on his legs.
The father threatens to sue you and says 'how dare you think that about me, I love my son!'
Which one of the following would be the most appropriate next step in management?

X A Admit the child to remove him from the possibly dangerous environment.

Call the police.


* QB
X Qc Ask the parents if there has been any abuse.

X D Speak to the wife privately about possible child abuse.

QE Report the family to child protective services.

Option E is correct

Once a healthcare worker forms an opinion, on reasonable grounds, that child abuse has occurred or is in
progress, reporting to Child Protection Service is mandatory. The physician is legally protected if the case is
found out not to be due child abuse, if reporting has occurred in good faith.

In this case, with several episodes of injuries with unfitting accounts, child abuse is very likely and the family
should be reported to the Child Protection Service immediately.

NOTE - The power of removing the child from the parents is not within the physician’s authority. This is
undertaken by authorities such as child protection services or courts of law.
(Option A ) Admitting the child for protection would be unnecessary as the child can be satisfactorily safeguarded
while in hospital.

(Option B ) Calling the police would have been indicated if the assault is in progress, which is not the case here.

(Options C and D ) When the belief of child is formed, talking to the parents would be incorrect as it is unlikely to
change the course of action. Abusive parents are not likely to give the exact account of the event and admit to
child abuse, nor are they likely to change their behavior without intervention.
Question ID 908

Title Medicolegal approach to a doctor with known HIV positive status

Notes
Q Flag as important

Font Size: A A A

One of your patients is a 37-year - old doctor, who has just recently found out he is HIV
positive. You are the only one that knows about this. Which one of the following you are
legally obliged to inform?

o His hospital administrator.

B Medical Board.

C His patients.

D His patients only if he performs surgeries where transmission is possible.

E No one without his consent.


X QB Medical Board.

X Qc His patients.

X QD His patients only if he performs surgeries where transmission is possible.

QE No one without his consent.

Option E is correct

Healthcare workers (HCWs), who are HIV positive, have a right to privacy as long as they are not posing others
at the risk of the infection. Therefore, you cannot inform anyone of his condition without his consent if he is not
posing others at risk.

(Options A , B and C ) There are guidelines and protocols for HCWs with blood-borne viruses, and it is assumed
that they follow these guidelines and precautionary measures as a part of their job. Neither the treating physician,
nor the patient is obliged to inform any authorities including Medical Board, state government, the insurance
company or his employer.

(Option D ) An HCW is not legally obliged to inform his/her patients of his/her HIV positivity. The risk of
transmission of the infection to the patients is extremely rare, especially when the physician takes precautionary
measures and/or is under treatment with antiretroviral therapy. However, HCWs should understand their
obligation to report their infections with blood-borne viruses (BBV ) status if required under jurisdictional legislation
and should be informed of relevant policies. They should understand their obligation to report all sharp injuries,
whether or not there was a risk of patient exposure.

TOPIC REVIEW
The following should be considered for HCWs with BBVs:

• All HCWs infected with a BBV should remain under regular medical supervision.
• HCWs must not perform EPPs (exposure-prone procedures) if they are human immunodeficiency virus
(HIV ) antibody positive.

• HCWs must not perform EPPs while they are hepatitis C virus (HCV) RNA positive but may be permitted to
return to EPPs after successful treatment or following spontaneous clearing of HCV RNA.

• HCWs must not perform EPPs while they are HBV DNA positive, but may be permitted to return to EPPs
following spontaneous clearing of HBV DNA or clearing of HBV DNA in response to treatment.
Question ID 876

Title Consent for ECT on a 52-year - old woman with depression and psychotic features

Notes
Q Flag as important

Font Size: A A A

A 52-year - old woman is involuntarily admitted to the psychiatric ward due to major
depression with psychotic features. She refuses to take anything by mouth including her
medications because she believes that she will die if she eats. Electroconvulsive therapy ( ECT)
has been decided for her as a life- saving measure and explained to her but she refuses to
consent to this treatment. She is clinically stable for now. Which one of the following is the
next best step in her management?

o Take consent from the Mental Health Tribunal.

B Give ECT without consent under duty of care.

C Obtain consent from her husband.

D Give her nasogastric feeding and anti- depressants.

E Obtain consent from the hospital administrator.


QA Take consent from the Mental Health Tribunal.

X B Give ECT without consent under duty of care.

Obtain consent from her husband.


* Oc

x Give her nasogastric feeding and anti- depressants.


OD

Obtain consent from the hospital administrator.


X O

Option A is correct

ECT is widely used as a treatment option. In Australia, the most frequent indication for ECT has been major
depression, especially if associated with psychotic features ( such as in this patient). Regulations regarding when
to use ECT and how to obtain consent vary from state to state.

For voluntary patients who have adequate capacity to make decisions for themselves, ECT can be
administered by authorized physicians if:

• The procedure and techniques have been fully explained to the patient
• All associated discomforts and potential adverse effects have been explained to the patient
• Alternative treatments (if available) has been explained to the patient
• The patient has been offered to ask any question regarding the procedure
• The patient is aware of his/her right to withdraw consent and discontinue the procedure at any given time
• The patient has been noted that she/she can have legal and medical advice before giving consent
-
NOTE Consent to ECT should be in writing.
For involuntary ECT, consent process and authorization varies in different states. ECT is different from other
procedures in terms of substitute decision making processes. In South Australia , for example, ECT should be
authorized by the Guardianship Board; however, should emergency arise, ECT can be given without applying the
case to the Guardianship Board. In the Australian Capital Territory and New South Wales, Mental Health Tribunal
is the authorized body for approval of ECT on involuntary patients.

Despite differences among different Australian states (and for the exam purposes) the following rules Produced
by Australian Health Ministers Advisory Council ( AHMAC ) can be applied in general:

The Tribunal may approve the performance of electro-convulsive therapy upon a person who is an involuntary
patient, a forensic patient, a patient under supervision or is subject to a community treatment order but it must not
give its approval unless satisfied that:
Despite differences among different Australian states (and for the exam purposes) the following rules Produced
by Australian Health Ministers Advisory Council ( AHMAC ) can be applied in general:

The Tribunal may approve the performance of electro-convulsive therapy upon a person who is an involuntary
patient, a forensic patient, a patient under supervision or is subject to a community treatment order but it must not
give its approval unless satisfied that:

• The person is not capable of giving informed consent


AND

• Two medical practitioners (at least one a psychiatrist) have formed the opinion after considering the
person’s clinical condition, history of treatment and any appropriate alternative treatments that electro-
convulsive therapy is reasonable and a proper treatment to administer for a the person and that without
that treatment the person is likely to suffer serious mental or physical deterioration.

Electro-convulsive therapy in life saving emergencies:


The authorized psychiatrist may authorize the performance of electro-convulsive therapy upon a person who is
an involuntary patient, a person under supervision or a forensic patient without having obtained the approval of a
tribunal if the authorized psychiatrist has the opinion that electroconvulsive therapy is necessary to save life of a
person or to prevent the person from suffering irreparable harm.

The authorized psychiatrist must report electroconvulsive therapy to the tribunal after it is performed.

In simple words, involuntary patients in an emergency condition, where delay can lead to serious harm to
them or others, are treated with ETC by an authorized psychiatrist without any need for approval from Mental
Health Tribunal, Guardianship Board, etc.
Where the situation is not likely to result in serious harm to the patient or others, the decision as to whether ECT
is performed as involuntary treatment should come from authorities.

It is strongly recommended that emergency ECT be avoided if possible. It is also strongly recommended that
emergency ECT is not given without a second opinion. If practical, the matter should be discussed with the chief
psychiatrist or his/her authorized representative before proceeding.

Impaired judgment and false bizarre beliefs of this patient, along with the major depression is indicative of major
depression with psychotic features, which is one of the most common and well-known indications for ECT as a
highly effective treatment . This patient , who clearly lacks decision-making capacity, is subject to involuntary
treatment after approval from Mental Health Tribunal.

(Option B ) Since the patient is clinically stable, no emergency situation is present to mandate ECT without
approval from the aforementioned authority.

(Options C and E ) ECT widely is different from other medical procedures where consent from relatives of a
patient who is subject to involuntary treatment can be obtained; therefore, consent from other people such as
relatives, hospital administrator, another colleague, a medical senior, etc. are incorrect options.

(Option D ) Nasogastric tube to feed or give medications does not eliminate the need for ECT where it is clearly
and necessarily indicated.

Only medical practitioners are permitted to provide ECT, and it must be performed in a hospital approved for this
purpose, whether public or private. A minimum of two medical practitioners must be present, of whom one should
be experienced in the administration of ECT and the other in anesthesia.
Question ID 874

Title Consent for ECT on a 48-year - old lady with depression and psychotic features

Notes
Q Flag as important

Font Size: A A A

You are the on -call psychiatrist on a night shift when you are informed by the nurse of the
psychiatry ward that a 48 -year - old lady with severe depression has refused to eat or drink for
the past two days. She believes she does not have bowels. She was brought to the hospital by
her son. On examination, she is severely dehydrated, but refuses to receive any oral intake.
Electroconvulsive therapy ( ETC) is considered for her by you as an authorized psychiatrist for
ECT, but she refuses to give consent to the procedure. Which one of the following is the most
appropriate next step in her management?

o Take consent for ECT from her son.

B Take consent from the Mental Health Tribunal.

C Take consent from hospital administrator.

0 Treat her with ECT without consent under duty of care.

E Give her nasogastric feeding and anti- depressants.


*o
Take consent for ECT from her son.

X QB Take consent from the Mental Health Tribunal.

X c Take consent from hospital administrator.

Treat her with ECT without consent under duty of care.


O

x o Give her nasogastric feeding and anti- depressants.

Option D is correct

ECT is widely used as a treatment option. In Australia, the most frequent indication for ECT has been major
depression, especially if associated with psychotic features (such as in this patient). Regulations regarding when
to use ECT and how to obtain consent vary from state to state.

For voluntary patients who have adequate capacity to make decisions for themselves, ECT can be
administered by authorized physicians if:

• The procedure and techniques have been fully explained to the patient
• All associated discomforts and potential adverse effects have been explained to the patient
• Alternative treatments (if available) has been explained to the patient
• The patient has been offered to ask any question regarding the procedure
• The patient is aware of his/her right to withdraw consent and discontinue the procedure at any given time
• The patient has been noted that she/she can have legal and medical advice before giving consent
-
NOTE Consent to ECT should be in writing.
For involuntary ECT, consent process and authorization varies in different states. ECT is different from other
procedures in terms of substitute decision making processes. In South Australia , for example, ECT should be
authorized by the Guardianship Board; however, should emergency arise, ECT can be given without applying the
case to the Guardianship Board. In the Australian Capital Territory and New South Wales, Mental Health Tribunal
is the authorized body for approval of ECT on involuntary patients.
19:27 W wary patients who have adequate capacity to make decisions for themselves, ECT can be
^
administered by authorized physicians if:
<§> ID
56% (
-
• The procedure and techniques have been fully explained to the patient
• All associated discomforts and potential adverse effects have been explained to the patient
• Alternative treatments (if available) has been explained to the patient
• The patient has been offered to ask any question regarding the procedure
• The patient is aware of his/her right to withdraw consent and discontinue the procedure at any given time
• The patient has been noted that she/she can have legal and medical advice before giving consent
NOTE - Consent to ECT should be in writing.
For involuntary ECT, consent process and authorization varies in different states. ECT is different from other
procedures in terms of substitute decision making processes. In South Australia, for example, ECT should be
authorized by the Guardianship Board; however, should emergency arise, ECT can be given without applying the
case to the Guardianship Board. In the Australian Capital Territory and New South Wales, Mental Health Tribunal
is the authorized body for approval of ECT on involuntary patients.

Despite differences among different Australian states (and for the exam purposes) the following rules Produced
by Australian Health Ministers Advisory Council ( AHMAC ) can be applied in general:

The Tribunal may approve the performance of electro-convulsive therapy upon a person who is an involuntary
patient, a forensic patient, a patient under supervision or is subject to a community treatment order but it must not
give its approval unless satisfied that:

• The person is not capable of giving informed consent


AND

• Two medical practitioners (at least one a psychiatrist) have formed the opinion after considering the
person’s clinical condition, history of treatment and any appropriate alternative treatments that electro-
convulsive therapy is reasonable and a proper treatment to administer for a the person and that without
that treatment the person is likely to suffer serious mental or physical deterioration.

Electro-convulsive therapy in life saving emergencies:


The authorized psychiatrist may authorize the performance of electro-convulsive therapy upon a person who is
an involuntary patient , a person under supervision or a forensic patient without having obtained the approval of a
tribunal if the authorized psychiatrist has the opinion that electroconvulsive therapy is necessary to save life of a
person or to prevent the person from suffering irreparable harm.

The authorized psychiatrist must report electroconvulsive therapy to the tribunal after it is performed.

In simple words, involuntary patients in an emergency condition, where delay can lead to serious harm to
them or others, are treated with ETC by an authorized psychiatrist without any need for approval from Mental
Health Tribunal, Guardianship Board, etc.
Where the situation is not likely to result in serious harm to the patient or others, the decision as to whether ECT
is performed as involuntary treatment should come from authorities .

It is strongly recommended that emergency ECT be avoided if possible. It is also strongly recommended that
emergency ECT is not given without a second opinion. If practical, the matter should be discussed with the chief
psychiatrist or his/her authorized representative before proceeding.

This patient is suffering from severe dehydrated that can be life threatening or at least pose substantial risk to her
health; therefore, she is in an emergency. Under this circumstancees , ECT should be proceeded with without
consent and under the duty of care as per the above.

Again, it is of paramount importance that physicians seek advice regarding the state legislations in practice.
Question ID 847

Title Appropriate respond to smoking request from an aggressive 26-year - old man

Notes
Q Flag as important

Font Size: A A A

A 26-year- old immigrant man stole a car and while on the run he hits a woman on the
road. Eventually, he ended up hitting the guardrail in a highway and injuring himself. He is in
the hospital now and behaves aggressively and rude. He requires orthopedic attention and
care. While in the ward, he insists to smoke a cigarette but smoking is not allowed in the ward.
He warns you, as his treating doctor, that he will sue you because he knows many influential
people and will make you lose your job. Which one of the following is the most appropriate
action in this situation?

o Discharge him because he is not cooperative.

B Contact immigration authorities to take over for legal proceeding.

C Tell him that you can only give him nicotine gum or patch.

0 Call the police to arrest him and take over.

E Take him to a safe place and let him smoke under supervision.
X QA Discharge him because he is not cooperative.

X QB Contact immigration authorities to take over for legal proceeding.

X c Tell him that you can only give him nicotine gum or patch.

X QD Call the police to arrest him and take over.

O Take him to a safe place and let him smoke under supervision.

Option E is correct

In mental health setting , cigarettes have been used as a patient management tool by staff, mediating exchanges
and relationships between staff and patients and between patients. Examples include using control over supply of
cigarettes to patients to comply with requests such as taking medications, getting dressed, agreeing to speak to
the treating doctor, etc. Although it has been a place of debate if implementation of non-smoking policies in
psychiatric wards is of benefit, it is still in practice.

In this situation though ( emergency setting ) where a severely agitated and disturbed patient is approached,
allowing him to smoke may help in de-escalation of the patient without unnecessary coercive treatments. A
common practice in Australia, when such a situation arises, is to allow the patient to smoke under appropriate
supervision and in place that the risk of others being exposed to cigarette smoke is nil or at least minimum.

Discharging an uncooperative agitated patient not only is inconsistent with the duty of care, but it also may pose
the patient and others at significant risk; therefore, not an appropriate action.

Discharging the patient ( option A ) or surrendering him while he is in need of medical care both for his psychiatric
and orthopedic problems to the immigration office ( option B) or the police (option D ) is not appropriate . The
police can deal with the patient, if necessary, after adequate care has been taken medically.

Nicotine gums and patches (option C ) may be helpful in reduction of agitation in patients who are quitting
smoking, but unlikely to satisfy this patient.
Question ID 840

Title Management of an elderly woman with advanced directive in an emergency situation

Notes
Q Flag as important

Font Size: A A A

A 73 -year - old woman has a living will clearly mentioning that she does not wish to be
admitted if she is terminally ill. Today, she is brought to the Emergency Department after she
sustained a fall at home and had a femoral neck fracture. In the emergency department, she
becomes drowsy after a morphine shot is given to her for pain control. Regarding her will,
which one of the following would be the next best step in management?

o Arrange for transferring to the operating room for surgical fixation of the fracture.

B Arrange a family meeting.

C Admit her.

D Refer her for palliative care.

E Check the validity of her will.


A 73-year - old woman has a living will clearly mentioning that she does not wish to be
admitted if she is terminally ill. Today, she is brought to the Emergency Department after she
sustained a fall at home and had a femoral neck fracture. In the emergency department, she
becomes drowsy after a morphine shot is given to her for pain control. Regarding her will,
which one of the following would be the next best step in management?

x QA Arrange for transferring to the operating room for surgical fixation of the fracture.

X B Arrange a family meeting.

Qc Admit her.

X QD Refer her for palliative care.

Check the validity of her will.


X O

Option C is correct

Based on the concept of autonomy, every individual has every right over their body if they are competent or
capacitated. In circumstances where the patient lacks competence to decide, a previous instruction by him/her
such as a valid living will or advance directives will guide the treating team as to treatment.

This patient has a living will that she should not be admitted if she is terminally ill. Her decision then should be
respected and acted upon if such circumstance arises, but as neither femoral neck fracture nor a completely
reversible adverse effect of opiates (drowsiness) is concordant with definition of a terminal illness, she should be
admitted for treatment of opiate overdose and reversal of the current condition. Once she is out of this state,
further management plan including fixation of her fracture by surgery or other measures can be discussed with
her.
Question ID 834

Title Appropriate management of situation with an Aboriginal mother

Notes
Q Flag as important

Font Size: A A A

An Indigenous woman brings her four - month- old boy for vaccination. During the interview,
she does not make any eye contacts and avoids conversations. When you ask her to hold the
child for you, so that you can inject the vaccine she denies. Which one of the following would
be the most appropriate management?

o Send a nurse for a home visit.

B Check her behavior in the next visit.

C Ask one of her family members to accompany her in the next visit.

D Call the Child Protection Service.

E Involve her in a parental program.


An Indigenous woman brings her four - month- old boy for vaccination. During the interview,
she does not make any eye contacts and avoids conversations. When you ask her to hold the
child for you, so that you can inject the vaccine she denies. Which one of the following would
be the most appropriate management?

Send a nurse for a home visit.


X O

X QB Check her behavior in the next visit.

y C Ask one of her family members to accompany her in the next visit.

X QD Call the Child Protection Service.

X o Involve her in a parental program.


Option C is correct

In visiting indigenous patients, cultural differences should always be borne in mind. These differences sometimes
are a significant barrier in establishing appropriate communication and rapport with the patient.

In some cultures, any relationship between people of opposite sex is considered a taboo and unethical. In some
cases, outsiders cannot easily be trusted, and therefore it is recommended that an Aboriginal health worker be
involved when assistance is required with a cultural issue. Here, there are some pointers towards the failure in
communication between the doctor and the patient. Firstly, she avoids eye contact. Second, she is reluctant to
converse and finally she refuses to hold the child for vaccination because when you get close to the baby you
may get close to her and, by this, breach her circle of safety.

In situations like this, the best step can be involvement of another family member in next visits to make the
patient feel more comfortable .

Thus far, no concern regarding child abuse or neglect has risen, because the mother has brought her child for
vaccination and this indicates that the mother cares about the baby and his safety. For this reason, calling the
child protection authorities or sending a nurse for a home visit would not be necessary.

The patient’s behavior is not likely to change in the next visit if no active measures are taken.

A summary of communication tips for dealing with indigenous people are as follow:

• Do not assume English is a first language, particularly in remote areas.


• Do not assume a nod means understanding and/or agreement to treatment.
• Check hearing because it might have been impaired due to chronic ear infection.
• Appreciate the different family network, particularly the tendency of grandmothers and aunts to care for
children.
• Do not assume a broken appointment means the patient will not return for treatment. Often family and
cultural duties take precedence.
• Be aware of cultural sensitivities.
• Do not touch a patient, particularly of the opposite sex, without seeking permission and explaining what
you are doing.
• Be aware that patients may not be comfortable with direct questions about their family and health.
• Do not be to stern or authoritative during a consultation.
• Ensure receptionists and other staff understand the cultural sensitivities of Indigenous patients.
• Be accepting, respecting and non-judgmental.
Question ID 818

Title Consent for treatment of a medical problem in an involuntarily admitted patient

Notes
Q Flag as important

Font Size: A A A

A 25 -year - old man is involuntarily admitted to the mental health ward for treatment of
psychosis. On physical examination, he is found to have a 5 x 5 cm ulcer on the plantar surface
of his left foot, requiring wound debridment, irrigation and intravenous antibiotics. Which one
of the following is the most appropriate approach regarding consent for treatment of his
wound?

o No consent is required as the patient is already involuntarily admitted.

B Take consent from the treating psychiatrist.

C Take consent from the patient.

Contact the patient ' s family for informed consent.

E Apply to the mental health tribunal for consent for wound debridment.
A 25-year - old man is involuntarily admitted to the mental health ward for treatment of
psychosis. On physical examination, he is found to have a 5 x5 cm ulcer on the plantar surface
of his left foot, requiring wound debridment, irrigation and intravenous antibiotics. Which one
of the following is the most appropriate approach regarding consent for treatment of his
wound?

No consent is required as the patient is already involuntarily admitted.


X O

X QB Take consent from the treating psychiatrist.

Qc Take consent from the patient.

X QD Contact the patient ' s family for informed consent.

X o Apply to the mental health tribunal for consent for wound debridment.

Option C is correct

Patients who are subject to involuntary treatment under mental health act, may still have capacity to give or
decline consent for a specific healthcare matter not related to their current mental illness (for example the use of
antibiotics for chest infection, or a surgical procedure on the limb). Assessment of the patient’s capacity to make
decisions about every other treatment other than that for the initial problem should be performed in the standard
way and documented appropriately.

If a patient lacks capacity to give consent to health care for a condition unrelated to the current mental illness, the
consent should be sought from a substitute decision maker in the same order as for other patients who are not
legally able to give informed consent. Involuntary treatment order only applies to the current mental health issue
and does not cover other areas of treatment.
Question ID 814

Title Management of a confused patient with suspected meningitis

Notes
Q Flag as important

Font Size: A A A

A 34-year- old man is brought to the Emergency Department with fever, headache and a
change in mental status leading to significant disorientation. A head CT scan is obtained which
is normal. Based on the diagnosis of meningitis, he is planned to be started on intravenous
antibiotics after a lumbar puncture is performed. The patient is agitated and is fighting with
anyone who tries to get near him. Some of his friends from work accompany him. Every time
an LP is about to be attempted, the patient pushes away the LP needle. Which one of the
following would be the most appropriate action to take in this situation?

o Sedate the patient and perform the LP.

B Wait for his relatives to arrive for consent.

C Use blood cultures as an alternative.

Arrange for a brain MRI.

E Ask his co -workers to sign the consent form.



4 w i i i .—
A r f t

A 34-year - old man is brought to the Emergency Department with fever, headache and a
change in mental status leading to significant disorientation. A head CT scan is obtained which
is normal. Based on the diagnosis of meningitis, he is planned to be started on intravenous
antibiotics after a lumbar puncture is performed. The patient is agitated and is fighting with
anyone who tries to get near him. Some of his friends from work accompany him. Every time
an LP is about to be attempted, the patient pushes away the LP needle. Which one of the
following would be the most appropriate action to take in this situation?

y QA Sedate the patient and perform the LP.

x QB Wait for his relatives to arrive for consent.

Use blood cultures as an alternative.


* Oc

X D Arrange for a brain MRI.

x OE Ask his co -workers to sign the consent form.

Option A is correct

The scenario describes a patient who is unable to give either an informed consent or informed refusal to the
procedure. He does not have the capacity to understand his medical condition and the consequences of
deferring the LP or antibiotics. On the other hand, there is no valid substitute decision maker (proxy ) or family
member to make decisions on the patient’s behalf. Under such circumstances, when there is an urgent life-
threatening or even severely painful medical condition and the patient is not competent to give consent to or
refuse the treatment and there is no substitute decision maker or family member, the patient ’s best interest will
guide the management.

Since performing an LP followed by intravenous antibiotics is the most important and essential step in
management of suspected meningitis, which is potentially life-threatening, the patient should be sedated, and
undergo LP.

(Option B ) Waiting for the relatives to arrive for consent before starting the treatment is not appropriate as
untreated meningitis carries significant risk of morbidity and mortality.

(Options C and D ) MRI or blood culture is not accurate for diagnosis and guidance of further treatment of
meningitis.

(Option E) Co-workers are not qualified to consent on the patient’s behalf.


Question 717
ID

Title Appropriate action in response to abortion request from the mother of an intellectually-
disabled female

Notes
Q Flag as important

Font Size: A A A

A mother brings her 18-year - old mentally retarded daughter at eight weeks pregnancy. The
mother requests abortion for her daughter as she is unable to look after herself. The patient
does not want an abortion. Which one of the following is the most appropriate action to take?

o Perform medical abortion as it is close to nine weeks; otherwise she will require
surgical abortion.

B Do not perform abortion as the patient did not consent.

C Refer the matter to Family Court.

Get consent from the local council.

E Discuss the matter with the father of the girl in a week.


A mother brings her 18-year - old mentally retarded daughter at eight weeks pregnancy. The
mother requests abortion for her daughter as she is unable to look after herself. The patient
does not want an abortion. Which one of the following is the most appropriate action to take?

Perform medical abortion as it is close to nine weeks; otherwise she will require
X O surgical abortion.

x QB Do not perform abortion as the patient did not consent.

y Q Refer the matter to Family Court.

X QD Get consent from the local council.

Discuss the matter with the father of the girl in a week.


X O
Option C is correct

Because of mental retardation, this patient does not have capacity to make decisions and give consent. Although
the mother can give consent on her behalf , but there are particular procedures for which no relative, no matter
how close, can give consent on the patient’s behalf and the case must be referred to either the Supreme Court or
the Family Court. In Australia, these two courts can exercise their role as the supreme parents of children.

An application to the court should be considered in situations so serious that neither the incompetent young
person, nor the parents or guardian can give valid consent to. These situations are as follow:

• The procedure is very high risk (for example, separating conjoined twins).
• There may be life-changing effects such as in:
• Sterilization of mentally disabled young persons
• Abortions
• Removal of life support
• Removal of organs for transplants
• Gender re-assignment
• Bone marrow harvest
• There is a strong objection from a dissenting parent
• A child with capacity to make decisions is refusing healthcare and there is significant risk of harm in them
doing so
• The procedure involves invasive, irreversible ( oophorectomy ) or major surgery. Life-saving emergency
surgeries are exceptions

Abortion in individuals who are incompetent due to mental disability should be consented to by the Supreme
Court or the Family court. Neither the patient, nor the carer consent is not valid for this procedure.
(Option A ) Performing the abortion without a court order is illegal and inappropriate.

(Option B ) The refusal of an incompetent patient does not eliminate the need for further action while the carer ’s
concerns have not been addressed yet.

(Option D ) Local councils are not authorized to make decision on this issue. As mentioned earlier, the decision is
in the power of the Supreme Court or Family Court.

(Option E) Like the mother, the father of the child is not authorized to make decisions in this regard.
Question ID 716

Title Appropriate response to a patient' s refusal of intimate physical examination

Notes
Q Flag as important

Font Size: A A A

Sue, an 18 year old girl, presents to your clinic with lower abdominal pain and fishy-smelling
vaginal discharge for the past three days. You are a male doctor and a female nurse is on duty
at the clinic. You mention to the patient that she needs a vaginal examination, but
she declines to be examined by you and says that she wants a female doctor to examine her.
Which one of the following is the most appropriate action in this situation?

o Call her boyfriend to be present while you are examining her.

B Treat the patient without vaginal examination.

C Convince the patient that female chaperon will be present during the examination.

D Defer the examination and inform the patient about the associated risks of not
doing the examination.

E Ask the nurse to perform the vaginal examination.


X QB Treat the patient without vaginal examination.

X Oc Convince the patient that female chaperon will be present during the examination.

QD Defer the examination and inform the patient about the associated risks of not
doing the examination.

X E Ask the nurse to perform the vaginal examination.

Option D is correct

Informed consent is required before an intimate examination is carried out on a patient. Intimate examinations
include examination of the breasts, genitalia and anus/rectum. Patients have a right to decline examination as
long as the decision is informed.

This patient does not want to be examined by a male doctor; therefore, her wish should be respected, and the
examination is deferred, but she must also be informed about how delayed treatment might affect her health and
alter the course of treatment.

Patients may ask for the presence of a chaperon, but it is not the patient’s concern here. The chaperon or a
female nurse can be present during the examination, but they cannot do the examination on your behalf. The
patient is unwilling to be examined by a physician of opposite sex and presence of a female nurse or chaperon
does not fulfill her wish. A patient may ask for a particular chaperon to be present or a particular health
practitioner ( maybe gender-based) to undertake the examination. Such requests are complied with where
possible.

While the patient is uncomfortable with being examined by a male doctor, presence of his boyfriend ( option A ) or
a nurse or chaperon (option C ) is incorrect. She does not want you to perform the examination; so, her wish
should be respected and any attempt to convince her otherwise is avoided.

(Option B ) Treating without appropriate investigation and just based on a speculated diagnosis is not an
acceptable option.

(Option E) A nurse cannot perform the examination on your behalf as it is your responsibility to treat her based
on your own direct findings during the exam .

-
NOTE It is wise to have a chaperon, nurse, or observer of the same sex in general present when performing
intimate examination, particularly when examining a patient of the opposite sex. If the patient refuses another
health practitioner to be present during the examination, this should be documented. Having a staff member
within your hearing is an advisable practice as a protective measure against possible accusations.
Question ID 715

Title Responding to the request of ' withholding information' by a patient' s relative

Notes
Q Flag as important

Font Size: A A A

After a course of full investigation on a 67- year - old woman, the diagnosis of pancreatic cancer
is established. You, as the treating doctor, are approaching the patient's room to break the
news to her when the patient's son steps forward and asks you to not tell her mother about
the diagnosis. He argues that his mother is very fragile at the moment and telling her about
the diagnosis will make her worse. You assess the patient's file. The consent form does not
have any instructions regarding the patient's wish as to whether she wants to be informed of
the diagnosis. Which one of the following is the most appropriate action to take?

o Tell the son that your decision cannot be based on the relatives' recommendations.

B Do not inform the patient of the diagnosis as it is not mentioned in the file whether
she wants to be informed of the diagnosis.

C Arrange a family meeting to further discuss the issue.

D Refer the case to the Guardianship Court.

E Withhold the information from the patient as it may lead to an emotional


breakdown.
y QA Tell the son that your decision cannot be based on the relatives' recommendations.

QB Do not inform the patient of the diagnosis as it is not mentioned in the file whether
* she wants to be informed of the diagnosis.

X Qc Arrange a family meeting to further discuss the issue .

X QD Refer the case to the Guardianship Court .

X E Withhold the information from the patient as it may lead to an emotional


breakdown.

Option A is correct

It is a patient’s right to know what you know and as soon as you know, and no recommendation from relatives, no
matter how rational it may sound , can breach this indisputable fact. The only exception is when the patient clearly
states that he/she does not want to be informed of the diagnosis.

There are very limited circumstances in which information may be deliberately withheld from a patient. This is
frequently referred to as ‘therapeutic privilege’. Particular information may be withheld where the practitioner (and
not the relatives ) believes, on reasonable grounds, that giving the information to the patient may damage the
patient ’s health. The responsibility is on the practitioner to show that providing the information would be
reasonably likely to cause significant harm. It is not acceptable to argue that if the risks associated with a
procedure or condition were disclosed, the patient may choose not to go ahead with it.

(Option B ) It is always assumed that the patient has the right and wants to use the right of being informed of the
diagnosis and actively participate in management plan. It should never be assumed otherwise unless the patient
clearly asks it.

(Option C ) Arranging a family meeting is futile because the patient should be informed regardless of the meeting
conclusion.

( Option D ) Referring the case to the Guardianship court is not appropriate because the rules are quite clear in
this regard. The Guardianship court will confirm your decision.

(Option E ) Fearing of the patient's emotional or physical reactions to bad news can never be an excuse
for withholding information from them.
Question 714
ID

Title Approach to conflicting wishes of the relatives of an incompetent patient regarding


treatment

Notes
Q Flag as important

Font Size: A A A

A 75-year- old man is brought to the hospital with reduced level of consciousness. CT scan of
the head shows epidural hematoma requiring non-urgent surgery. You are unable to discuss
the procedure with the patient due to his decreased level of consciousness. His spouse and
the eldest son are present. His spouse says that her husband never wanted to come to the
hospital and would never want to be resuscitated or have any surgery on him. His son
requests you to do everything possible to save his father's life. Which one of the following is
the most appropriate action?

o Listen to his son.

B Arrange a family meeting to reach a consensus.

C Apply for guardianship.

D Listen to his wife.

E Request the medical superintendent to assess the patient capacity.


A 75 -year - old man is brought to the hospital with reduced level of consciousness. CT scan of
the head shows epidural hematoma requiring non-urgent surgery. You are unable to discuss
the procedure with the patient due to his decreased level of consciousness. His spouse and
the eldest son are present. His spouse says that her husband never wanted to come to the
hospital and would never want to be resuscitated or have any surgery on him. His son
requests you to do everything possible to save his father's life. Which one of the following is
the most appropriate action?

*o
Listen to his son.

X QB Arrange a family meeting to reach a consensus.

X c Apply for guardianship.

QD Listen to his wife.

X OE Request the medical superintendent to assess the patient capacity.


Option D is correct

If a person cannot give consent for their own treatment and there is no advance health directive present, a
health practitioner should obtain consent from the "person responsible". In this situation, patient' spouse
can consent for the patient's future treatment. A spouse's opinion is considered more valid and appropriate
than that of sons or daughters; therefore, the directives should be obtained from the wife. One other
important step is trying to find out whether the patient was competent or not when he expressed such
wishes.

In instances where no advance health directive or a spouse, family members, or carer is present, a medical
superintendent or authorized medical officer can decide for life-saving emergency surgery if the patient is
incompetent to make a decision.

Application to Guardianship is required for all those patients who lack the capacity to decide about their
treatment; however it usually takes few days.

Under the Guardianship Act 1987, a ' person responsible' can make decisions about most medical or dental
treatments.

A ' person responsible' is, in order of priority:


(1) - The legally appointed guardian of the person (including enduring guardian) with the function of
consenting to medical/ dental treatment

If there is not one then

(2) - Their spouse or de facto spouse or same sex partner, or if there is no spouse or de facto spouse or
same sex partner

If there is not one then

(3) - Their unpaid carer

If there is not one then

(4) - The patient's nearest relative over the age of 18 year, which means ( in order of preference):

• Son or daughter
• Father or mother
• Brother or sister (including adopted people and 'step' relationships)
• Grandfather or grandmother
• Grandson or granddaughter
• Uncle or aunt
• Nephew or niece
When there are two relatives in the same position (for example, a brother and a sister) the elder will be the
person responsible.
Question ID 713

Title Consent in a 17-year -old boy with confusion and a perforated appendicitis

Notes
Q Flag as important

Font Size: A A A

A 17 -year - old boy is admitted to the emergency department due to progressive right lower
quadrant abdominal pain. The patient is febrile and confused. Investigation establishes the
diagnosis of a perforated appendicitis. He is booked for emergency laparotomy. His 18-year -
old sister accompanies her. Which one of the following is correct regarding the consent for the
surgery?

The consent should be obtained from the patient.


OA

B The consent should be obtained from the sister.

C Two doctors are required to give consent.

D The consent should be obtained from the parents.

E There is no need for the consent as the case is an emergency.


A 17-year - old boy is admitted to the emergency department due to progressive right lower
quadrant abdominal pain. The patient is febrile and confused. Investigation establishes the
diagnosis of a perforated appendicitis. He is booked for emergency laparotomy. His 18-year -
old sister accompanies her. Which one of the following is correct regarding the consent for the
surgery?

X QA The consent should be obtained from the patient.

QB The consent should be obtained from the sister.

X Qc Two doctors are required to give consent.

X QD The consent should be obtained from the parents.

x There is no need for the consent as the case is an emergency.


OE
Option B is correct

The age at which an individual is considered an adult is 18 years old, but when it comes to medical treatment,
one can give consent a younger age, provided that they have a good understanding of the condition, treatment
options and potential consequences. The age at which a minor can consent for medical treatment is 14 or 16,
depending on the state. The validity of consent of a minor is based on the Gillick competence rather than the age.
If this patient was alert and mentally competent, the consent could be obtained from him, but since he is
confused, he is incompetent for this purpose.

In cases where there is an emergency and the patient is either underage to give consent, or is a consenting adult
but is mentally incapacitate due to any reason, and medical treatment is required to save their life or prevent
serious damage to their health, treatment may be provided without consent if there is no advance directives or
substitute decision maker available (guardian, adult relative, etc.). In common law it is referred to as ' defense of
necessity’.

The treatment provided must be required urgently and not be given just because it is convenient. It must be
proportionate to the patient’s needs. Treatment that goes beyond what is necessary to avert the crisis should not
be given.

Importantly, treatment cannot be provided where the patient has made a valid advance directive or refusal of
treatment certificate that clearly says they have refused the intervention, providing that certain conditions are met.

Perforated appendicitis is a genuine emergency putting the patient’s health at serious risk if surgery is not
performed immediately ; however, since his adult relative (her sister) is available , consent should be obtained
from her as her substitute decision maker.

(Option A ) This patient is 17 years old and would have been capable of giving consent if he was alert and
seemed to understand his condition , the proposed treatment options and their consequences, but his confusion
precludes him from such an understanding at present. He is temporarily incompetent to give informed consent.

(Option C ) Since an adult relative as the patient’s substitute decision maker is available, consent should be
sought from her. It is important to note that the substitute decision maker should act based on the patient’s best
interest, which in this case is surgery. If she refuses the surgery, the doctor should not comply with her wish
because it is not in the patient’s best interest. If such conflicts arise, the doctor can proceed to the surgery while
following the guidelines of the state he/ she is practicing. In South Australia , for example, the doctor can proceed
with the treatment if another physician who has visited the patient in person confirms (in writing) the need for
emergency intervention. In other states different rules have been put in place .

(Option D ) Parents of the boy were the most appropriate options from whom to obtain consent if they were
available. Even an option suggesting obtaining consent from the parents over the phone could be the most
appropriate one; however, the question does not mention if the parents could be available in person or by
telephone immediately; therefore, his next of kin who is currently available is the most appropriate option.

(Option E) Consent is not required in cases of genuine emergencies, where the patient cannot consent, there is
not advance directive, and no substitute decision maker is available, or if the substitute decision maker refuses a
treatment that is in the patient’s best interest.
Question ID 639

Title Management of a situation between a mother and her eight-year - old son

Notes
Q Flag as important

Font Size: A A A

Mother of an eight -year - old boy brings him to your practice with complaints about his
behavior. She believes that her son has recently become rude and does not listen to her. She
sometimes beats him and locks him up in a room as a punishment for his behavior. The
mother also mentions that she has become irritable recently and cannot sleep as usual. Which
one of the following is the most appropriate step in management?

o Talk to the boy.

B Notify the Child Protection Service.

C Refer the mother to Good Parenting programs.

D Inform the police about the child abuse.

E Refer the mother for psychiatric counselling.


.
Will JILL .

Mother of an eight -year - old boy brings him to your practice with complaints about his
behavior. She believes that her son has recently become rude and does not listen to her. She
sometimes beats him and locks him up in a room as a punishment for his behavior. The
mother also mentions that she has become irritable recently and cannot sleep as usual. Which
one of the following is the most appropriate step in management?

X A Talk to the boy.

QB Notify the Child Protection Service.

X Qc Refer the mother to Good Parenting programs.

X QD Inform the police about the child abuse.

Refer the mother for psychiatric counselling.


*O
Family violence is coercive and controlling behavior by a family member that causes physical, sexual and /or
emotional damage to others in the family, causing them to live in fear or being threatened. Family violence is
most commonly committed by one partner towards another (‘domestic violence’ or ‘ intimate partner abuse’)
and/or by an adult towards a child or children. Other forms include elder abuse or sibling abuse. Any kind of
abuse may have long-term detrimental effects.

There is a legal obligation on both medical and nursing staff to notify Child Protection Service if they have
formed the opinion that a child is need of protection. The child in need for protection is defined by ‘ Children
Youth and Families Act 2005 - SECT 162 PART 4.1’:
(1) For the purposes of this Act a child needs protection if any of the following grounds exist:

(a) The child has been abandoned by his or her parents and after reasonable inquiries-

i. the parents cannot be found; and

ii. no other suitable person can be found who is willing and able to care for the child;

(b ) The child's parents are dead or incapacitated and there is no other suitable person willing and able to
care for the child;

(c) The child has suffered, or is likely to suffer, significant harm as a result of physical injury and the child's
parents have not protected, or are unlikely to protect, the child from harm of that type;

(d) The child has suffered, or is likely to suffer, significant harm as a result of sexual abuse and the child' s
parents have not protected, or are unlikely to protect, the child from harm of that type;

(e ) The child has suffered, or is likely to suffer, emotional or psychological harm of such a kind that the
child's emotional or intellectual development is, or is likely to be, significantly damaged and the child' s
parents have not protected, or are unlikely to protect, the child from harm of that type;

(f) The child's physical development or health has been, or is likely to be, significantly harmed and the
child's parents have not provided, arranged or allowed the provision of, or are unlikely to provide, arrange
or allow the provision of, basic care or effective medical, surgical or other remedial care.

(2) For the purposes of subsections (1)(c) to (1)(f), the harm may be constituted by a single act , omission or
circumstance or accumulate through a series of acts, omissions or circumstances.

The way the mother is using to punish the child for his behavior is likely to pose risks to both his physical and
emotional well-being. According to the above, the mother 's action should be reported to child protection
authorities.

Reporting child abuse of any kind is mandatory for all medical practitioners and must be reported to authorities as
soon as possible (e.g. , Child Protection Service ). In this case, it is even more paramount because the mother has
the symptoms of a psychiatric problem.

Talking to the boy for obtaining collateral history or referring the mother for counselling and good parenting
program may be considered later in the course of action; however, securing the child’s safety, is of significance
priority.

In this scenario, contacting the police directly would not be necessary. Calling the police may be justified in cases
of in emergency situations, but not here.
Question ID 607

Title Management of a trauma patient who refuses emergency blood transfusion

Notes
Q Flag as important

Font Size: A A A

A 49-year- old married man is brought to the Emergency Department with internal bleeding from
injuries sustained in a motor vehicle crash. The patient has been stabilized but requires blood
transfusion prior to planned emergency surgery. When he is asked to give consent for the
transfusion, he states that blood transfusion is forbidden in his religion . Which one of the following
is the most appropriate next step in management?

o Assess the patient's decision- making capacity to refuse the transfusion.

B Call the patient's wife to obtain consent for the transfusion.

C Call the Guardianship Court for consent.

D Proceed with transfusion against the patient's wish.

E Talk to the patient about blood substitutes and ask him if he would like to consider
them.
A 49-year- old married man is brought to the Emergency Department with internal bleeding from
injuries sustained in a motor vehicle crash. The patient has been stabilized but requires blood
transfusion prior to planned emergency surgery. When he is asked to give consent for the
transfusion, he states that blood transfusion is forbidden in his religion. Which one of the following
is the most appropriate next step in management?

o Assess the patient's decision- making capacity to refuse the transfusion.

X QB Call the patient's wife to obtain consent for the transfusion.

X c Call the Guardianship Court for consent.

X QD Proceed with transfusion against the patient's wish.

*o
Talk to the patient about blood substitutes and ask him if he would like to consider
them.

Option A is correct

According to Australian law every competent adult individual has full control over his/ her body even if this
means refusing life - saving treatment. There is no exception to this rule if the patient is competent;
therefore, the first step would be assessing the patient's competency to make decisions. Decision-making
capacity includes understanding, appreciation and reasoning.

Once the patient is proved to be competent, other measures such as transfusing Hemacil ® or other
intravenous fluids may be considered and discussed with the patient.

It should be borne in mind that although parents of an underage individual are required to consent to
the treatment, this will not apply if the treatment is life-saving or limb -saving. In other words, if the
patient in the vignette was a minor, blood transfusion should be performed against the parent's will.
Guardianship court is consulted when patient lacks competency either due to impaired judgment or
suppressed level of consciousness and there is no advance directives and there is conflict between family
members and the treating physician, or between the family members themselves.
Question ID 606

Title Approach to a ' Do not Resuscitate' request by a critically ill patient

Notes
Q Flag as important

Font Size: A A A

A 67 -year - old man presents to the emergency department with severe chest pain. An ECG is
consistent with inferior myocardial infarction. Initial management with aspirin, oxygen, and
morphine is done. While arrangements are made to transfer the patient for percutaneous
coronary intervention, he becomes pulseless and unconscious. Cardiac monitoring shows
ventricular fibrillation. He is immediately managed with defibrillation and regains his
consciousness and pulse again. He then asks you to not resuscitate him again if anything
happened to him. Which one of the following is the most appropriate approach?

o Assess his mental competence.

B Write "do not resuscitate" order in his file.

C Overrule his request as this decision is irrational.

0 Ask his wife to convince him otherwise.

E Refer the case to the Guardianship Court.


A 67-year- old man presents to the emergency department with severe chest pain. An ECG is
consistent with inferior myocardial infarction. Initial management with aspirin, oxygen, and
morphine is done. While arrangements are made to transfer the patient for percutaneous
coronary intervention, he becomes pulseless and unconscious. Cardiac monitoring shows
ventricular fibrillation. He is immediately managed with defibrillation and regains his
consciousness and pulse again. He then asks you to not resuscitate him again if anything
happened to him. Which one of the following is the most appropriate approach?

/ QA Assess his mental competence.

x QB Write "do not resuscitate" order in his file.

Qc Overrule his request as this decision is irrational.


*
X QD Ask his wife to convince him otherwise.

X QE Refer the case to the Guardianship Court.

Option A is correct

Every competent adult patient has full control over his/her body. They may refuse a medical treatment even
though it is life-saving. They may ask for withholding or withdrawal of treatment, including cardiopulmonary
resuscitation. When such requests are faced, the next best step is assessing the patient’s mental competence.
Once the patient is found competent, their wish should be respected and followed.

(Option B) A ‘do not resuscitate’ order can be put in place after he is found to be mentally competent and it is
assured that he is fully informed of the consequences.

(Option C) By overruling a competent patient’s wish and doing the resuscitation, the crime of battery is
committed, and the perpetrator may face criminal charges.

(Option D) While a competent adult patient can make decisions for himself/herself asking his/her relatives to
convince them otherwise is not correct. This is an issue related to them and they need to address it between
themselves at their discretion.

(Option E ) The rules and regulations on this issue are quite clear and straightforward and there is no need to
involve the Guardianship court or any other authorities.
Question ID 605

Title The responsible person for obtaining informed consent from a patient undergoing surgery

Notes
Q Flag as important

Font Size: A A A

You are a medical officer in a tertiary hospital. A patient is about to undergo abdominal
surgery. When the patient is being transferred to the operating theatre, the nurse informs you
that the consent for the surgery has not yet been obtained. Which one of the following is the
most appropriate step regarding consent?

o Send the patient to the operating room and ask the anesthesiologist to obtain the
consent.

B Obtain the consent yourself and send the patient for the surgery.

C Download information from the internet and discuss it with the patient and
obtain consent.

D Call the treating surgeon and ask him to obtain consent.

E Ask the attending nurse to obtain consent from the patient on his way to the
operating theatre.
You are a medical officer in a tertiary hospital. A patient is about to undergo abdominal
surgery. When the patient is being transferred to the operating theatre, the nurse informs you
that the consent for the surgery has not yet been obtained. Which one of the following is the
most appropriate step regarding consent?

X A Send the patient to the operating room and ask the anesthesiologist to obtain the
consent.

X QB Obtain the consent yourself and send the patient for the surgery.

Download information from the internet and discuss it with the patient and
X Oc obtain consent.

S QD Call
^e treating surgeon and ask him to obtain consent.
Ask the attending nurse to obtain consent from the patient on his way to the
X O
operating theatre.

Option D is correct

The ultimate responsibility of obtaining consent is with the clinician directly in charge of the treatment;
therefore, the operating surgeon should be called for obtaining consent from the patient. However, it is
possible for that clinician to delegate authority to other healthcare professionals,such as more junior
members of staff. The proviso for such delegation is that the person obtaining consent must be fully
equipped to deal with the consent process.

Reference( s) • Australian Medico - legal Handbook - Elsevier Australia ( 2008) - page 68


Question ID 424

Title Advice regarding cervical cancer screening for a 26-year-old lesbian

Notes
Q Flag as important

Font Size: A A A

A 26-year - old female comes for cervical cancer screening by HPV testing. She never had sexual
activity with a man, but is a lesbian and has a girlfriend. She prefers to see a lesbian -friendly
health care provider for her test. Which one of the following would be the most appropriate
action to take in this situation?

o She does not need cervical cancer screening.

B Refer her to a lesbian- friendly clinic in the area.

C Do the test yourself and then refer her to a gynecologist.

D Ask your clinic nurse to do the test in your clinic.

E Advise her to follow safe sex practice principles even with a female partner.
19:34 Fri 24 Apr -r >
(S 57% HU'

X A She does not need cervical cancer screening.

B Refer her to a lesbian-friendly clinic in the area.

X c Do the test yourself and then refer her to a gynecologist.

X D Ask your clinic nurse to do the test in your clinic.

Advise her to follow safe sex practice principles even with a female partner.
X O

Option B is correct

Specific subtypes of human papilloma viruses (HPV), most commonly types 16. 18, 31, 33, 35, are associated
with premalignant and cancerous cervical lesions. HPV can spread through sexual intercourse , as well as skin-to-
skin contact in female-to-female sexual relationship. Therefore, lesbians need pap smears like heterosexual
women.

There is no evidence to suggest that HPV infection rate is lower in lesbians, and rates of cervical abnormalities
for lesbians are like those of heterosexual women.

As this patient prefers to see a lesbian-friendly healthcare provider, the next best step would be encouraging her
for screening and referring her to a lesbian-friendly clinic in your area. Failing to do so may result in the patient
not seeking screening. This not only poses the patient at risk; it may also lead to lawsuit for the physician once
the patient contracts cervical cancer.

(Option A ) Lesbians need cervical cancer screening like heterosexual women. Telling her that she does not
require pap smears is not correct.

(Option C ) Performing a cervical cancer screening test without the patient's consent is an act of battery and
should be avoided.

(Option D ) It is a wise practice to have a female nurse or chaperon present while the doctor takes sample for
HPV testing, particularly if the doctor is male, but taking samples is the responsibility of doctors not nurses.

(Option E) Advising precautionary measures such as safe sex practice is appropriate but does not eliminate the
need for screening tests as the most important issue here.

Reference( s) • Murtagh's General Practice - McGraw Hill - 5th Edition - pages 929 and 932

• Good Medical Practice - Professionalism Ethics and Law - Cambridge - page 165
Question ID 99

Title Management of a family crisis caused by pathological gambling

Notes
Q Flag as important

Font Size: A A A

A 42-year - old woman comes to your clinic complaining of headache and asking for a sick leave
certificate. She has separated two years ago and currently lives with her two children Sarah
and Beth, 7 and 4 years old respectively. She admits to smoking cigarettes and using alcohol
on a regular basis. During the examination, she looks depressed. More probing causes her to
break in tears and confess that she cannot resist gambling. She has lost all of her money and
cannot provide care for her children anymore. Which one of the following would be the most
appropriate next step in management?

o Refer her to specialist gambling treatment bodies (e.g., gamblers anonymous).

B Inform the Child Protection Service.

C Arrange for financial support.

Arrange for cognitive behavior therapy.

E Perform motivational interview.


A 42-year - old woman comes to your clinic complaining of headache and asking for a sick leave
certificate. She has separated two years ago and currently lives with her two children Sarah
and Beth, 7 and 4 years old respectively. She admits to smoking cigarettes and using alcohol
on a regular basis. During the examination, she looks depressed. More probing causes her to
break in tears and confess that she cannot resist gambling. She has lost all of her money and
cannot provide care for her children anymore. Which one of the following would be the most
appropriate next step in management?

X OA Refer her to specialist gambling treatment bodies (e.g., gamblers anonymous).

QB Inform the Child Protection Service.

X Qc Arrange for financial support.

X QD Arrange for cognitive behavior therapy.

Perform motivational interview.


X O

Option B is correct

This patient has features of problem gambling requiring help and support, but the main issue and the most
important step would be the two little children she has back at home. Pathological gambling is not normally
associated with violence against children; rather the abuse tends to exist in the form of ‘neglect’.

This woman’s problematic behavior has posed her children at risk; therefore, the next step in management would
be protecting the children from the harm threatening them by informing child protection authorities.

(Options A and E ) Once the children’s safety is ensured, attention should be turned to the patient. Motivational
interview would be the most important initial step, followed by referral to specialist gambling treatment agencies if
the patient is willing or has persuaded so.

(Option C) Options suggesting ‘arranging for financial support, local funding-for-poor programs, etc.’ are
incorrect because such measures, if necessary, will be taken by authorized agencies after the patient has been
referred to them.

(Option D ) Cognitive behavioral therapy or other methods such as cognitive therapy or brief interventions can be
applied when felt necessary.

You might also like